Содержание

Тест по физике на тему «прямолинейное равномерное и равноускоренное движение». Прямолинейное равномерное движение Тс 1 перемещение скорость равномерное прямолинейное движение

Равномерное движение – это движение с постоянной скоростью, то есть когда скорость не изменяется (v = const) и ускорения или замедления не происходит (а = 0).

Прямолинейное движение – это движение по прямой линии, то есть траектория прямолинейного движения – это прямая линия.

Равномерное прямолинейное движение – это движение, при котором тело за любые равные промежутки времени совершает одинаковые перемещения. Например, если мы разобьём какой-то временной интервал на отрезки по одной секунде, то при равномерном движении тело будет перемещаться на одинаковое расстояние за каждый из этих отрезков времени.

Скорость равномерного прямолинейного движения не зависит от времени и в каждой точке траектории направлена также, как и перемещение тела. То есть вектор перемещения совпадает по направлению с вектором скорости. При этом средняя скорость за любой промежуток времени равна мгновенной скорости:

V cp = v

Пройденный путь

при прямолинейном движении равен модулю перемещения. Если положительное направление оси ОХ совпадает с направлением движения, то проекция скорости на ось ОХ равна величине скорости и положительна:

V x = v, то есть v > 0

Проекция перемещения на ось ОХ равна:

S = vt = x – x 0

где x 0 – начальная координата тела, х – конечная координата тела (или координата тела в любой момент времени)

Уравнение движения , то есть зависимость координаты тела от времени х = х(t), принимает вид:

Х = x 0 + vt

Если положительное направление оси ОХ противоположно направлению движения тела, то проекция скорости тела на ось ОХ отрицательна, скорость меньше нуля (v

Х = x 0 — vt

Зависимость скорости, координат и пути от времени

Зависимость проекции скорости тела от времени показана на рис. 1.11. Так как скорость постоянна (v = const), то графиком скорости является прямая линия, параллельная оси времени Ot.

Рис. 1.11. Зависимость проекции скорости тела от времени при равномерном прямолинейном движении.

Проекция перемещения на координатную ось численно равна площади прямоугольника ОАВС (рис. 1.12), так как величина вектора перемещения равна произведению вектора скорости на время, за которое было совершено перемещение.

Рис. 1.12. Зависимость проекции перемещения тела от времени при равномерном прямолинейном движении.

График зависимости перемещения от времени показан на рис. 1.13. Из графика видно, что проекция скорости равна

V = s 1 / t 1 = tg α

где α – угол наклона графика к оси времени.Чем больше угол α, тем быстрее движется тело, то есть тем больше его скорость (больший путь тело проходит за меньшее время). Тангенс угла наклона касательной к графику зависимости координаты от времени равен скорости:

Tg α = v

Рис. 1.13. Зависимость проекции перемещения тела от времени при равномерном прямолинейном движении.

Зависимость координаты от времени показана на рис. 1.14. Из рисунка видно, что

Tg α 1 > tg α 2

следовательно, скорость тела 1 выше скорости тела 2 (v 1 > v 2).

Tg α 3 = v 3

Если тело покоится, то графиком координаты является прямая, параллельная оси времени, то есть

Х = х 0

Рис. 1.14. Зависимость координаты тела от времени при равномерном прямолинейном движении.

Данное пособие включает тренировочные задания. тесты для самоконтроля, самостоятельные работы, контрольные работы и примеры решения типовых задач. Предлагаемые дидактические материалы составлены в полном соответствии со структурой и методологией учебника А. В. Перышкина, К. М. Гутник «Физика. 9 класс».

ТЗ-1. Путь и перемещение .
1. Укажите, в каком из приведенных ниже примеров тело можно считать материальной точкой:
а) Земля, движущаяся вокруг Солнца;
б) Земля, вращающаяся вокруг своей оси;
в) Луна, вращающаяся вокруг Земли;
г) Луна, на поверхности которой движется луноход;
д) молот, брошенный спортсменом;

е) спортивный молот, который изготавливают на станке.
2. Что определяет пассажир автобуса по цифрам на километровых столбах, установленных вдоль шоссе, — перемещение или пройденный автобусом путь?
3. На рисунке 1 изображены траектории полета снарядов. Равны ли для этих движений пройденные снарядами пути? перемещения?
4. Тело, брошенное вертикально вверх из точки Л, упало в шахту (рис. 2). Чему равны пройденный телом путь и модуль перемещения, если АВ = 15 м, ВС — 18 м?
5. Спортсмену предстоит пробежать один круг (400 м). Чему равен модуль перемещения, если он: а) пробежал 200 м пути; б) финишировал? Дорожку стадиона считать окружностью.
6. Белка бежит внутри колеса, находясь на одной и той же высоте относительно пола. Равны ли путь и перемещение при таком движении?

Предисловие.
ТРЕНИРОВОЧНЫЕ ЗАДАНИЯ
ТЗ-1. Путь и перемещение.
ТЗ-2. Прямолинейное равномерное движение.
ТЗ-3. Относительность движения.
ТЗ-4. Прямолинейное равноускоренное движение.

ТЗ-5. Законы Ньютона.
ТЗ-6. Свободное падение тел.
ТЗ-7. Закон всемирного тяготения. Движение тела
ТЗ-8.Импульс тела. Закон сохранения импульса.
Закон сохранения энергии.
ТЗ-9. Механические колебания и волны. Звук.
ТЗ-10. Электромагнитное поле.
ТЗ-11. Строение атома и атомного ядра.
ТЕСТЫ ДЛЯ САМОКОНТРОЛЯ
ТС-1. Прямолинейное равномерное движение.
ТС-2. Прямолинейное равноускоренное движение.
ТС-3. Законы Ньютона.
ТС-4. Свободное падение тел.
ТС-5. Закон всемирного тяготения. Движение тела
по окружности. Искусственные спутники Земли..
ТС-6. Импульс тела. Закон сохранения импульса.
Закон сохранения энергии.
ТС-7. Механические колебания.
ТС-8. Механические волны. Звук.
ТС-9. Электромагнитное поле.
ТС-10. Строение атома и атомного ядра.
САМОСТОЯТЕЛЬНЫЕ РАБОТЫ
СР-1. Путь и перемещение.
СР-2. Прямолинейное равномерное движение.
СР-3. Прямолинейное равномерное движение.
Графические задачи.
СР-4. Относительность движения.
СР-5. Прямолинейное равноускоренное движение..
СР-6. Прямолинейное равноускоренное движение.
Графические задачи.
СР-7. Законы Ньютона.
СР-8. Свободное падение тел.
СР-9. Закон всемирного тяготения.
Искусственные спутники Земли.
СР-10. Движение тела по окружности.
СР-11. Импульс тела. Закон сохранения импульса.
Закон сохранения энергии.
СР-12. Механические колебания.
СР-13. Механические волны. Звук.
СР-14. Электромагнитное поле.
СР-15. Строение атома и атомного ядра.
КОНТРОЛЬНЫЕ РАБОТЫ
КР-1. Прямолинейное равноускоренное движение.
КР-2. Законы Ньютона.
КР-3. Закон всемирного тяготения. Движение тела
по окружности. Искусственные спутники Земли.
КР-4. Закон сохранения импульса.
Закон сохранения энергии.
КР-5. Механические колебания и волны.
КР-6. Электромагнитное поле.
ПРИМЕРЫ РЕШЕНИЯ ТИПОВЫХ ЗАДАЧ

Законы взаимодействия и движения тел.
Механические колебания и волны.
Электромагнитное поле.
ОТВЕТЫ
Тренировочные задания.
Тесты для самоконтроля.
Самостоятельные работы.
Контрольные работы.
Список литературы.

Бесплатно скачать электронную книгу в удобном формате, смотреть и читать:
Скачать книгу Физика, 9 класс, учебно-методическое пособие, Марон А.Е., Марон Е.А., 2014 — fileskachat.com, быстрое и бесплатное скачивание.

Скачать pdf
Ниже можно купить эту книгу по лучшей цене со скидкой с доставкой по всей России.

Существуют различные виды механического движения. В зависимости от формы траектории движение может быть прямолинейным или криволинейным. При движении скорость тела может оставаться постоянной или с течение времени изменяться. В зависимости от характера изменения скорости движение будет равномерным или неравномерным.

Прямолинейное движение – это движение, при котором траектория тела (точки) – прямая линия. Например, движение автомобиля по участку дороги, на котором нет подъемов, спусков, поворотов.

Равномерным прямолинейным движением называют движение, при котором тело за любые равные промежутки времени проходит одинаковые пути и направление движения не меняетс я.

Если сравнить равномерное движение нескольких тел, то можно отметить, что быстрота изменения их положения в пространстве может быть различной, что характеризуется физической величиной, которая называется скоростью.

Скоростью равномерного прямолинейного движения называют векторную физическую величину, равную отношению перемещения тела ко времени, за которое это перемещение произошло.

(1)

Единица скорости в СИ – метр в секунду (1м/ c ). За единицу скорости принимают скорость такого равномерного движения, при котором тело за 1 с совершает перемещение 1м .

При прямолинейном равномерном движении скорость не изменяется с течением времени.

Зная скорость равномерного движения, можно найти перемещение тела за любой промежуток времени:

(2)

При равномерном прямолинейном движении векторы скорости и перемещения направлены в одну сторону.

Основной задачей механики является определение положение тела в любой момент времени, то есть определение его координат. Уравнение движения – это зависимость координаты тела от времени при равномерном прямолинейном движении.

Тело совершило перемещение . Направим координатную осьX по направлению перемещения тела. x 0 – начальная координата тела, x – конечная координата тела.

Таким образом, координату тела при равномерном прямолинейном движении в любой момент времени можно определить, если известны его начальная координата и проекция скорости движения на ось Х . Проекции скорости и перемещения могут быть как положительными, так и отрицательными.

График зависимости модуля вектора скорости от времени при равномерном движении – это прямая, параллельная оси абсцисс. Действительно, с течением времени скорость при таком движении остается постоянной.

График зависимости скорости тела от времени при равномерном движении V=const

При прямолинейном равномерном движении модуль вектора перемещения численно равен площади под графиком перемещения к оси времени.

График зависимости перемещения тела, от времени при прямолинейном равномерном движении – это прямая, проходящая через начало координат. Причем чем круче проходит график перемещения, тем больше скорость движения тела.

График зависимости пути, пройденного телом, от времени

При прямолинейном равномерном движении модуль вектора скорости численно равен тангенсу угла наклона графика перемещения к оси времени.

Поскольку зависимость координаты тела от времени – линейная функция, то соответствующий график зависимости (график движения) представляет собой прямую линию. Пример построения такого графика показан на рисунке.

График зависимости координаты тела от времени

ТС -1 Прямолинейное равномерное движение.

I вариант .

1. Велосипедист, двигаясь равномерно, проезжает 20 м за 2 с. Определите, какой путь он проедет при движении с той же скоростью за 10 с.

А. 60 м. Б. 100 м. В. 150 м.

2
. На рисунке приведен график зависимости пути при движении велосипедиста от времени. Определите по этому графику путь, который проехал велосипедист в промежуток времени от 1 до 4 с.

3. По графику определите скорость движения велосипедиста в момент времени t =2 с.

4
. На рисунке представлены графики движения трех тел. Какое из этих тел движется с наибольшей по модулю скоростью в момент времени t =5 с?

5. По графику определите скорость движения первого тела в момент времени t =5 с.

А. 2 с, 5 м.

Б. 4 с, 10 м.

В. 5 с,15 м.

7. Запишите уравнение движения
второго тела по графику.

А.
.

Б.
.

В.
.


9. Катер плывет против течения реки. Какова скорость катера относительно берега, если скорость катера относительно воды 4 м/с, а скорость течения реки 3 м/с?

А. 7 м/с. Б. 5 м/с. В. 1 м/с.

10. Поезд прошел первые 40 км со скоростью 80 км/ч, а следующие 50 км – со скоростью 100 км/ч. Определите среднюю скорость поезда на всем пути.

А. 95 км/ч. Б. 85 км/ч. В. 90 км/ч.

ТС-1. Прямолинейное равномерное движение.

I
I
вариант.

    Автомобиль, двигаясь равномерно, проехал 50 м за 2с. Какой путь он проедет за 20 с, двигаясь с той же скоростью?

А
. 500 м. Б. 1000 м. В. 250 м.

2. Определите по графику зависимости пути от времени путь, пройденный телом за промежуток времени от 3 до 5 с.

    По графику определите скорость движения тела в момент времени t =4 с.

4
. На рисунке представлены графики движения трех тел. Какое из этих тел движется с наименьшей скоростью в момент времени t =2 с.

5. По графику движения определите скорость движения второго тела в момент времени 6 с.

6. По графику движения определите время и место встречи первого и второго тел.

А. 2 с, 10 м.

Б. 1 с, 5 м.

7. Запишите уравнение движения первого тела по графику.

А.
.

Б.
.

В.
.

8. Движение тела описывается уравнением
. На каком из графиков представлена зависимость координаты этого тела от времени?

9. Эскалатор метро движется вниз со скоростью 0,7 м/с. Какова скорость пассажира относительно земли, если он идет вверх со скоростью 0,7 м/с относительно эскалатора?

А.0 м/с. Б. 1,4 м/с. В. 1 м/с.

10. Автомобиль проехал первые 20 км со скоростью 50 км/ч, а следующие 60 км – со скоростью 100 км/ч. Определите среднюю скорость автомобиля на всем пути.

А. 90 км/ч. Б. 80 км/ч. В. 70 км/ч.

Тест с ответами по физике “Перемещение. Скорость. Равномерное прямолинейное движение” для 10 класса

1. Тело, брошенное под углом к горизонту, упало на землю на расстоянии 40 м от точки бросания. Максимальная высота подъема над землей в процессе движения составила 30 м. Определите модуль перемещения тела от точки бросания до точки падения на землю:
а) 40 м +
б) 30 м
в) 50 м

2. Какова скорость пешехода, преодолевающего ровным шагом расстояние 1,2 км за 20 мин:
а) 10 м/с
б) 1 м/с +
в) 2 км/ч

3. Конькобежец пробежал на стадионе 6 кругов радиусом 50 м. Определите модуль перемещения конькобежца:
а) 1884 м
б) 100 м
в) 0 м +

4. Найдите скорость (в м/с) равномерного в течение 1,5 мин полёта воздушного шара, за которые он пролетел 540 м:
а) 6 м/с +
б) 15 м/с
в) 54 м/с

5. Конькобежец пробежал на стадионе 4 круга радиусом 50 м. Определите пройденный конькобежцем путь:
а) 314 м
б) 100 м
в) 1256 м +

6. Какая принята единица скорости в СИ:
а) миллиметр в секунду (мм/с)
б) метр в секунду (м/с) +
в) сантиметр в секунду (см/с)

7. Турист прошел по горизонтальному полю 4 км строго на север, затем еще 3 км на восток. Найдите модуль перемещения туриста:
а) 4 км
б) 3 км
в) 5 км +

8. Скорость – это физическая величина, которая показывает:
а) какой путь проходит тело за единицу времени +
б) как быстро движется тело
в) пройденный телом путь

9. Турист прошел по горизонтальному полю 400 м строго на север, затем еще 300 м на восток. Найдите пройденный туристом путь:
а) 500 м
б) 300 м
в) 700 м +

10. Две автомашины движутся по дороге с постоянными скоростями 10 и 15 м/с. Начальное расстояние между машинами равно 1 км. Определите, за какое время вторая машина догонит первую:
а) 80 с
б) 200 с +
в) 50 с

11. Ракета поднялась на высоту 15 км и вернулась на Землю в точку старта. Определите пройденный ракетой путь:
а) 30 км +
б) 15 км
в) 7,5 км

12. Двигаясь равномерно, велосипедист проезжает 40 м за 4 с. Какой путь он проедет при движении с той же скоростью за 20 с:
а) 60 м
б) 200 м +
в) 30 м

13. Ракета поднялась на высоту 20 км и вернулась на Землю в точку старта. Определите модуль перемещения ракеты:
а) 0 км +
б) 20 км
в) 10 км

14. Колонна машин движется по шоссе со скоростью 10 м/с, растянувшись на расстояние 2 км. Из хвоста колонны выезжает мотоциклист со скоростью 20 м/с и движется к голове колонны. За какое время он достигнет головы колонны:
а) 30 с
б) 60 с
в) 200 с +

15. Как должно двигаться тело, чтобы пройденный путь был равен модулю перемещения:
а) по прямой, не и меняя направления движения +
б) по окружности
в) по кривой линии

16. Катер, двигаясь равномерно, проезжает 60 м за 2 с. Рассчитайте, какой путь он проедет за 10 с, двигаясь с той же скоростью:
а) 500 м
б) 300 м +
в) 100 м

17. Утром вы выходите из дома, а вечером снова возвращаетесь. Что больше: пройденный вами путь или модуль перемещения:
а) модуль перемещения
б) они равны
в) пройденный путь +

18. Самолёт совершает механическое движение:
а) если положение самолёта неизвестно
б) если изменяется положение самолёта относительно аэродрома +
в) если положение самолёта относительно аэродрома не изменяется

19. Что является траекторией движения молекулы воздуха:
а) дуга параболы
б) дуга окружности
в) ломаная линия +

20. Перпендикулярно падающие капли со скорости 8 м/с на окне равномерно движущегося автобуса под углом 60º к вертикали. Скорость движения автобуса равна:
а) 85 км/ч
б) 81 км/ч +
в) 88 км/ч

21. Автобус проехал свой маршрут от станции до конечной остановки за 0,8 ч со средней скоростью 32 км/ч. Чему равен его путь:
а) 25,6 км +
б) 32 км
в) 40 км

22. Автобус везёт пассажиров по прямой дороге со скоростью 10 м/с. Пассажир равномерно идёт по салону автобуса со скоростью 1 м/с относительно автобуса, двигаясь от задней двери к кабине водителя. Чему равен модуль скорости пассажира относительно дороги:
а) 19 м/с
б) 11 м/с +
в) 8 м/с

23. Самый быстрый пока серийный американский автомобиль разгоняется примерно до 430 км/ч. Считая его скорость равной 400 км/ч, рассчитайте, сколько времени ему понадобится, чтобы проехать 40 км:
а) 5 мин
б) ≈ 17 мин
в) 6 мин +

24. Велосипедист, двигаясь под уклон, проехал расстояние между двумя пунктами со скоростью, равной 15 км/ч. Обратно он ехал вдвое медленнее. Какова средняя путевая скорость на всем пути:
а) 9 км/ч
б) 10 км/ч +
в) 12 км/ч

25. Одна из моделей европейского автомобиля «Феррари» достигает скорости 360 км/ч. Какой путь пройдёт этот автомобиль за 10 мин с такой скоростью:
а) 10 км
б) 600 км
в) 60 км +

26. Вертолет равномерно поднимается вертикально вверх. Какова траектория крайней точки лопасти вертолета в системе отсчета, связанной с корпусом вертолета:
а) прямая
б) окружность +
в) точка

27. Определите среднюю скорость автомобиля на пути длиной 300 км. Первую половину этого пути он прошёл за 1,5 ч, а вторую за 2,5 ч:
а) 75 км/ч +
б) 60 км/ч
в) 40 км/ч

28. Пловец плывет по течению реки. Определите скорость пловца относительно берега, если скорость пловца относительно воды 0,4 м/с, а скорость течения реки 0,3 м/с:
а) 0,1 м/с
б) 0,5 м/с
в) 0,7 м/с +

29. Рассчитайте среднюю скорость движения (в м/с) поезда между двумя станциями, расстояние между которыми 30 км, если он, выйдя с одной станции, прибыл на другую через 30 мин:
а) ≈ 16,7 м/с +
б) ≈ 167 м/с
в) 10 м/с

30. Определите скорость (в м/с) равномерного скольжения конькобежца, который за 5 мин проехал путь длиной 1,5 км:
а) 3 м/с
б) 5 м/с +
в) 30 м/с

Тест. Система отсчета. Перемещение. Скорость равномерного прямолинейного движения

Будьте внимательны! У Вас есть 10 минут на прохождение теста. Система оценивания — 5 балльная. Разбалловка теста — 3,4,5 баллов, в зависимости от сложности вопроса. Порядок заданий и вариантов ответов в тесте случайный. С допущенными ошибками и верными ответами можно будет ознакомиться после прохождения теста. Удачи!

Список вопросов теста

Вопрос 1

В некоторой инерциальной системе отсчёта точка N имеет координаты (3;4). Найдите длину радиус-вектора, описывающего положение точки N.

Варианты ответов
Вопрос 2

Некоторый вектор составляет угол 150о с осью икс. Тогда, этот вектор будет…

Варианты ответов
  • Отрицательным
  • Положительным
  • Равен нулю
  • Либо отрицательным, либо положительным, но не равным нулю
Вопрос 3

Выберете верные утверждения

Варианты ответов
  • Радиус-вектор соединяет начало координат и данную точку пространства
  • Значение радиус-вектора не может быть отрицательным
  • Радиус-вектором можно пользоваться только в системе координат на плоскости
  • Длина радиус-вектора зависит от выбора начала координат
Вопрос 4

Для того, чтобы создать систему отсчета, необходимо иметь…

Варианты ответов
  • Систему координат
  • Радиус-вектор
  • Часы
  • Тело отсчета
  • Трехмерную систему координат
Вопрос 5

Модуль вектора а равен 10 вектор а перпендикулярен оси у. Тогда…

Варианты ответов
  • Проекция вектора а на ось х равна 0
  • Проекция вектора а на ось у равна 0
  • Проекция вектора а на ось х равна 10
  • Проекция вектора а на ось у равна 10
  • Вектор а находится в трехмерной системе координат
Вопрос 6

Выберете верные утверждения

Варианты ответов
  • Перемещение не может быть больше пройденного пути
  • Перемещение — это векторная величина
  • Пройденный путь — это векторная величина
  • Перемещение — это радиус-вектор
  • Пройденный путь — это радиус-вектор
Вопрос 7

Модуль вектора а равен 6 и вектор а составляет угол 30 градусов с осью х. Тогда…

Варианты ответов
  • Проекция вектора а на ось х равна 3
  • Проекция вектора а на ось у равна 3
  • Проекция вектора а на ось х равна 5,2
  • Проекция вектора а на ось у равна 5,2
Вопрос 8

Тело переместилось из точки M (4;3) в точку N (7;7). Найдите модуль перемещения данного тела.

Вопрос 9

Тело переместилось из точки M (2;4) в точку N (3;8) за 5 секунд. Найдите модуль скорости данного тела.

Вопрос 10

Тело переместилось из точки M (5;5) в точку N (8;7). Найдите угол (в градусах) между направлением вектора скорости и осью х. Ответ округлите до целого числа.

Дидактические материалы по физике 10. Дидактические материалы по физике (10 класс)

Тесты для самоконтроля
Самостоятельные работы
Разноуровневые контрольные работы
УДК 373.167.1:53 ББК 22.3я72 М28

Марон, А. Е.

М28 Физика. 10 класс: дидактические материалы / А. Е. Ма­рон* Е. А. Марон. — 2-е изд., стереотип. — М.: Дрофа, 2005. — 156, с. : ил.

ШВК 5-7107-9105-9

Данное пособие включает тесты для самоконтроля, самостоятельные работы, разноуровневые контрольные работы.

Предлагаемые дидактические материалы составлены в полном соответствии со структурой и методологией учебника В. А. Касьянова «Физика. 10 класс*.

УДК 373.167.1:63 ББК 22.3я72

ISBN 5-7107-9105-9

О ООО «Дрофа*, 2004

Предисловие

Предлагаемые дидактические материалы входят в учебно-методическое обеспечение образовательных про­грамм по физике, рекомендованных Министерством об­разования Российской Федерации для средней школы, и составлены в полном соответствии со структурой и мето­дологией учебника В. А. Касьянова «Физика. 10 класс».

Пособие включает тесты для самоконтроля (ТС), само­стоятельные работы (СР) и контрольные работы (КР).

Комплект предусматривает организацию всех основ­ных этапов учебно-познавательной деятельности школь­ников: применение и актуализацию теоретических зна­ний, самоконтроль качества усвоения материала, выпол­нение самостоятельных и контрольных работ.

Тесты для самоконтроля с выбором ответа предназна­чены для проведения оперативного поурочного тематиче­ского контроля и самоконтроля знаний. В зависимости от конкретных условий (подготовка класса, организация разноуровневого обучения и т. д.) учитель может варьи­ровать набор тестовых заданий и определять время их выполнения.

Самостоятельные работы содержат 5 вариантов и рас­считаны примерно на 20 минут каждая.

Контрольные работы являются тематическими. Они рассчитаны на один урок и составлены в четырех вариан­тах. Каждый вариант содержит блоки задач разных уров­ней сложности, которые отделены в пособии друг от дру­га чертой. Первый и второй уровни сложности (I и И) соответствуют требованиям к уровню подготовки вы­пускников средней школы, третий уровень (111) преду­сматривает углубленное изучение физики. Самостоятель­ные и разноуровневые контрольные работы, тесты для самоконтроля, включенные в общую систему организа­ции активной учебно-познавательной деятельности уча­щихся, позволяют сформировать такие важные качества личности, как активность, самостоятельность, самодиаг­ностика и самооценка учебных достижений.

Всего в комплекте содержится более 1000 задач и за­даний, к большинству из которых приведены ответы.

3

Тесты для Самоконтроля
ТС-1. Перемещение. Скорость. Равномерное прямолинейное движение

Вариант 1


  1. Двигаясь равномерно, велосипедист проезжает 40 м за 4 с. Какой путь он проедет при движении с той же скоро­стью за 20 с?
А. 30 м. Б. 50 м. В. 200 м.

  1. На рисунке 1 приведен график движения мотоциклис­та. Определите по графику путь, пройденный мотоцик­листом в промежуток времени от 2 до 4 с.
А. 6 м. Б. 2 м. В. 10 м.

  1. На рисунке 2 представлены графики движения трех тел. Какой из этих графиков соответствует движению с большей скоростью?
А. 1 . Б. 2. В. 3.

  1. По графику движения, представленному на рисунке 3, определите скорость тела.
А. 1 м/с. Б. 3 м/с. В. 9 м/с.

  1. Две автомашины движутся по дороге с постоянными скоростями 10 и 15 м/с. Начальное расстояние между машинами равно 1 км. Определите, за какое время вто­рая машина догонит первую.
А. 50 с. Б. 80 с. В. 200 с.

Вариант 2


  1. Катер, двигаясь равномерно, проезжает 60 м за 2 с. Рассчитайте, какой путь он проедет за 10 с, двигаясь с той же скоростью.
А. 300 м. Б. 500 м. В. 100 м.

4


X, м,

х, м

1

X, м

12



/.

2

8

8

/

0

1 3

Ь, с

С

0

Рис. 4

Рис. 5



  1. Определите по графику движения (рис. 4) путь, прой­денный автомобилем в промежуток времени от 1 до 3 с.
Л. 8 м. Б. 4 м. В. 12 м.

  1. На рисунке 5 представлены три графика движения. Какой из этих графиков соответствует движению с мень­шей скоростью?
А. 1 . Б. 2. В. 3.


  1. По графику движения (рис. 6 ) определите скорость тела.
А. 8 м/с. Б. 4 м/с. В. 2 м/с.

  1. Колонна машин движется по шоссе со скоростью 10 м/с, растянувшись на расстояние 2 км. Из хвоста ко­лонны выезжает мотоциклист со скоростью 20 м/с и дви­жется к голове колонны. За какое время он достигнет го­ловы колонны?
А. 200 с. Б. 60 с. В. 40 с.

ТС-2. Прямолинейное движение с постоянным ускорением

Вариант 1


  1. Определите, какой из графиков (рис. 7) соответствует равнозамедленному движению тела.
А. 1.
Б. 2.

В. 3.

2. По графику зависимости скорос­ти от времени (рис. 8 ) определите ус­корение тела.

А. 0,5 м/с2. В. 4 м/с2.

Б. 2 м/с2.

с

5

Vf

t, с

t, с

Рис. 10


  1. Определите, на каком из графиков (рис. 9) представле­но движение тела, имеющего наименьшее ускорение.
А. 1. Б .2. В. 3.


  1. По графику зависимости скорости автомобиля от вре­мени (рис. 10 ) определите перемещение автомобиля за первые 3 с его движения.
А. 60 м. Б. 90 м. В. 30 м.

  1. Тело движется без начальной скорости с ускорением

  1. 5 м/с2. Определите путь, пройденный телом за первую секунду.
А. 0,25 м. Б. 1 м. В. 0,5 м.

Вариант 2


  1. Определите, какой из графиков (рис. 11) соответствует равноускоренному движению тела.
А. 1. Б .2. В. 3.

  1. По графику зависимости скорости от времени (рис. 12) определите ускорение тела.
А. 5 м/с2. Б. 1 м/с2. В. 2 м/с2.



  1. Определите, на каком из графиков (рис. 13) представ­лено движение тела, имеющего наибольшее ускорение.
А . 1. Б. 2. В. 3.


  1. По графику зависимости скорости мотоциклиста от времени (рис. 14) определите перемещение мотоциклиста за первые 2 с его движения.
А. 40 м. Б. 30 м. В. 20 м.

  1. После старта гоночный автомобиль достиг скорости 360 км/ч за 25 с. Определите расстояние, пройденное ав­томобилем за это время.
А. 1250 м. Б. 1400 м. В. 1500 м.

ТС-3. Свободное падение. Баллистическое движение1

Вариант 1


  1. Чему равна скорость свободно падающего тела через

  1. с после начала падения, если и0 = 0 ?
А. 20 м/с. Б. 10 м/с. В. 30 м/с.

  1. С какой высоты был сброшен предмет, если он упал на землю через 2 с?
А. 30 м. Б. 20 м. В. 10 м.

  1. Рассчитайте время свободного падения тела с высоты 20 м.
А. 1с. Б. 3 с. В. 2 с.

  1. Тело брошено вертикально вверх со скоростью 30 м/с. Какова максимальная высота подъема тела?
А. 45 м. Б. 50 м. В. 90 м.
1 При решении задач принять Ј = 10 м/с2

7



  1. Мяч бросают с крыши, находящейся на высоте 20 м от
    поверхности земли. Его начальная скорость равна 25 м/с
    и направлена горизонтально. Чему равна дальность поле-
    та мяча по горизонтали?
А. 50 м. Б. 100 м. В. 75 м.

Вариант 2


  1. Определите скорость свободно падающего тела через

  1. с после начала падения, если и0 —
0 .

А. 10 м/с. Б. 30 м/с. В. 20 м/с.


  1. Какова глубина ущелья, если упавший в него камень
    коснулся дна через 4 с?
А. 80 м. Б. 100 м. В. 150 м.

  1. Мяч упал на землю с высоты 80 м. Определите, сколь-
    ко времени мяч находился в полете.
А. 2 с. Б. 1 с. В. 4 с.

  1. Стрела выпущена из лука вертикально вверх со скоро-
    стью 20 м/с. Рассчитайте максимальную высоту подъема
    стрелы.
А. 10 м. Б. 20 м. В. 30 м.

  1. Самолет летит горизонтально на высоте 8 км со скоро-
    стью 1800 км/ч. За сколько километров до цели летчик
    должен сбросить бомбу, чтобы поразить цель?
А. 40 км. Б. 20 км. В. 10 км.

ТС-4. Кинематика периодического движения

Вариант 1


  1. Тело движется равномерно по окружности против ча-
    совой стрелки (рис. 15). Как направлен вектор ускорения
при таком движении?

А.

1. В. 3.

Б.

2.

2. Тело движется по окружности радиусом


  1. м со скоростью Юл м/с. Определите пери-
    од вращения тела.
А. 0,8 с. В. 2 с.

Б. 1 с. Рис. 15

8


  1. Мотоциклист совершает поворот по круговой траекто­рии радиусом 50 м с постоянной по модулю скоростью 10 м/с. Каково ускорение мотоциклиста?
А. 1 м/с2. Б. 3 м/с2. В. 2 м/с2.

  1. Частица совершает гармонические колебания по зако­ну х =
10 cos nt/10 см. Определите координату частицы в момент времени t = 10 с.

А. 10 см. Б. -10 см. В. 0.


  1. t = 10 с.
А. 1 м/с. Б. 0. В. 2 м/с.

Вариант 2


  1. Тело движется равномерно по окруж-
    ности по часовой стрелке (рис. 16). Как
    направлен вектор ускорения при таком
    движении?
А. 1 . В. 3.

Б.

2.

  1. Какова частота вращения тела, движущегося по ок­ружности радиусом 5 м со скоростью 5 п м/с?
А. 2 Гц. Б. 0,5 Гц. В. 4 Гц.

  1. Трамвайный вагон движется на повороте по закругле­нию радиусом 40 м. Рассчитайте скорость трамвая, если центростремительное ускорение равно 0,4 м/с2.
А. 2 м/с. Б. 1 м/с. В. 4 м/с.

  1. Тело совершает гармонические колебания по закону х = 5 cos nt/6 см. Определите координату тела в момент времени Ј = 2 с.
А. 2,5 см. Б. 2 см. В. 0,4 см.

  1. По условию предыдущей задачи определите скорость частицы в момент времени t = 6 с.
А. 0. Б. 1 м/с. В. 0,5 м/с.

Данное пособие включает тесты для самоконтроля, самостоятельные работы, разноуровневые контрольные работы.
Предлагаемые дидактические материалы составлены в полном соответствии со структурой и методологией учебников В. А. Касьянова «Физика. Базовый уровень. 10 класс» и «Физика. Углубленный уровень. 10 класс».

Примеры заданий:

ТС 1. Перемещение. Скорость.
Равномерное прямолинейное движение
Вариант 1
1. Двигаясь равномерно, велосипедист проезжает 40 м за 4 с. Какой путь он проедет при движении с той же скоростью за 20 с?
А. 30 м. Б. 50 м. В. 200 м.
2. На рисунке 1 приведен график движения мотоциклиста. Определите по графику путь, пройденный мотоциклистом в промежуток времени от 2 до 4 с.
А. 6м. Б. 2 м. В. 10 м.
3. На рисунке 2 представлены графики движения трех тел. Какой из этих графиков соответствует движению с большей скоростью?
А. 1. Б. 2. В. 3.
4. По графику движения, представленному на рисунке 3, определите скорость тела.
А. 1 м/с. Б. 3 м/с. В. 9 м/с.
5. Две автомашины движутся по дороге с постоянными скоростями 10 и 15 м/с. Начальное расстояние между машинами равно 1 км. Определите, за какое время вторая машина догонит первую.
А. 50 с. Б. 80 с. В. 200 с.

Предисловие.
ТЕСТЫ ДЛЯ САМОКОНТРОЛЯ
ТС-1. Перемещение. Скорость.
Равномерное прямолинейное движение.
ТС-2. Прямолинейное движение с постоянным ускорением
ТС-3. Свободное падение. Баллистическое движение.
ТС-4. Кинематика периодического движения.
ТС-5. Законы Ньютона.
ТС-6. Силы в механике.
ТС-7. Применение законов Ньютона.
ТС-8. Закон сохранения импульса.
ТС-9. Работа силы. Мощность.
ТС-10. Потенциальная и кинетическая энергия.
ТС-11. Закон сохранения механической энергии.
ТС-12. Движение тел в гравитационном поле.
ТС-13. Динамика свободных и вынужденных колебаний.
ТС-14. Релятивистская механика.
ТС-15. Молекулярная структура вещества.
ТС-16. Температура. Основное уравнение молекулярно-кинетической теории.
ТС-17. Уравнение Клапейрона-Менделеева. Изопроцессы.
ТС-18. Внутренняя энергия. Работа газа при изопроцессах. Первый закон термодинамики.
ТС-19. Тепловые двигатели.
ТС-20. Испарение и конденсация. Насыщенный пар. Влажность воздуха. Кипение жидкости.
ТС-21. Поверхностное натяжение. Смачивание, капиллярность.
ТС-22. Кристаллизация и плавление твердых тел.
ТС-23. Механические свойства твердых тел.
ТС-24. Механические и звуковые волны.
ТС-25. Закон сохранения заряда. Закон Кулона.
ТС-26. Напряженность электростатического поля.
ТС-27. Работа сил электростатического поля. Потенциал электростатического поля.
ТС-28. Диэлектрики и проводники в электростатическом поле.
ТС-29. Электроемкость уединенного проводника и конденсатора. Энергия электростатического поля.
САМОСТОЯТЕЛЬНЫЕ РАБОТЫ
СР-1. Равномерное прямолинейное движение.
СР-2. Прямолинейное движение с постоянным ускорением.
СР-3. Свободное падение. Баллистическое движение.
СР-4. Кинематика периодического движения.
СР-5. Законы Ньютона.
СР-6. Силы в механике.
СР-7. Применение законов Ньютона.
СР-8. Закон сохранения импульса.
СР-9. Работа силы. Мощность.
СР-9. Работа силы. Мощность.
СР-10. Потенциальная и кинетическая энергия. Закон сохранения энергии.
СР-11. Абсолютно неупругое и абсолютно упругое столкновение.
СР-12. Движение тел в гравитационном поле.
СР-13. Динамика свободных и вынужденных колебаний.
СР-14. Релятивистская механика.
СР-15. Молекулярная структура вещества.
СР-16. Температура. Основное уравнение молекулярно-кинетической теории.
СР-17. Уравнение Клапейрона-Менделеева. Изопроцессы.
СР-18. Внутренняя энергия. Работа газа при изопроцессах.
СР-19. Первый закон термодинамики.
СР-20. Тепловые двигатели.
СР-21. Испарение и конденсация. Насыщенный пар. Влажность воздуха.
СР-22. Поверхностное натяжение. Смачивание, капиллярность.
СР-23. Кристаллизация и плавление твердых тел. Механические свойства твердых тел.
СР-24. Механические и звуковые волны.
СР-25. Закон сохранения заряда. Закон Кулона.
СР-26. Напряженность электростатического поля.
СР-27. Работа сил электростатического поля. Потенциал.
СР-28. Диэлектрики и проводники в электростатическом поле.
СР-29. Электроемкость. Энергия электростатического поля
КОНТРОЛЬНЫЕ РАБОТЫ
КР-1. Прямолинейное движение.
КР-2. Свободное падение тел. Баллистическое движение.
КР-3. Кинематика периодического движения.
КР-4. Законы Ньютона.
КР-5. Применение законов Ньютона.
КР-6. Закон сохранения импульса.
КР-7. Закон сохранения энергии.
КР-8. Молекулярно-кинетическая теория идеального газа
КР-9. Термодинамика.
КР-10. Агрегатные состояния вещества.
КР-11. Механические и звуковые волны.
КР-12. Силы электромагнитного взаимодействия неподвижных зарядов.
КР-13. Энергия электромагнитного взаимодействия неподвижных зарядов.
ОТВЕТЫ
Тесты для самоконтроля.
Самостоятельные работы.
Контрольные работы.
Список литературы.

Бесплатно скачать электронную книгу в удобном формате, смотреть и читать:


Скачать книгу Физика, 10 класс, дидактические материалы к учебникам Касьянова В.А., Марон А.Е., 2014 — fileskachat.com, быстрое и бесплатное скачивание.
  • Физика, 10 класс, базовый уровень, учебник, Касьянов В.А., 2014

Данное пособие включает тесты для самоконтроля, самостоятельные работы, разноуровневые контрольные работы.
Предлагаемые дидактические материалы составлены в полном соответствии со структурой и методологией учебника В. А. Касьянова «Физика. 10 класс».

Примеры.
Самолет летит горизонтально на высоте 8 км со скоростью 1800 км/ч. За сколько километров до цели летчик должен сбросить бомбу, чтобы поразить цель?
А. 40 км.
В. 20 км.
В. 10 км.

Трамвайный вагон движется на повороте по закруглению радиусом 40 м. Рассчитайте скорость трамвая, если центростремительное ускорение равно 0,4 м/с2.
А. 2 м/с.
Б. 1 м/с.
В. 4 м/с.

Равнодействующая всех сил, действующих на тело, равна нулю. Движется ли это тело или находится в состоянии покоя?
A. Тело движется равномерно и прямолинейно или находится в состоянии покоя.
Б. Тело движется равномерно и прямолинейно.
B. Тело находится в состоянии покоя.

Содержание
Предисловие 3
ТЕСТЫ ДЛЯ САМОКОНТРОЛЯ
ТС-1. Перемещение. Скорость. Равномерное прямолинейное движение 4
ТС-2. Прямолинейное движение с постоянным ускорением 5
ТС-3. Свободное падение. Баллистическое движение 7
ТС-4. Кинематика периодического движения 8
ТС-5. Законы Ньютона 10
ТС-6. Силы в механике 11
ТС-7. Применение законов Ньютона 12
ТС-8. Закон сохранения импульса 14
ТС-9. Работа силы. Мощность 16
ТС-10. Потенциальная и кинетическая энергия 17
ТС-11. Закон сохранения механической энергии 18
ТС-12. Движение тел в гравитационном поле 20
ТС-13. Динамика свободных и вынужденных колебаний 22
ТС-14. Релятивистская механика 23
ТС-15. Молекулярная структура вещества 24
ТС-16. Температура. Основное уравнение молекулярно-кинетической теории 26
ТС-17. Уравнение Клапейрона-Менделеева. Изопроцессы 27
ТС-18. Внутренняя энергия. Работа газа при изопроцессах. Первый закон термодинамики 29
ТС-19. Тепловые двигатели 30
ТС-20. Испарение и конденсация. Насыщенный пар. Влажность воздуха. Кипение жидкости 32
ТС-21. Поверхностное натяжение. Смачивание, капиллярность 33
ТС-22. Кристаллизация и плавление твердых тел 35
ТС-23. Механические свойства твердых тел 37
ТС-24. Механические и звуковые волны 39
ТС-25. Закон сохранения заряда. Закон Кулона 40
ТС-26. Напряженность электростатического поля 42
ТС-27. Работа сил электростатического поля. Потенциал электростатического поля 44
ТС-28. Диэлектрики и проводники в электростатическом поле 47
ТС-29. Электроемкость уединенного проводника и конденсатора. Энергия электростатического поля 49
САМОСТОЯТЕЛЬНЫЕ РАБОТЫ
СР-1. Равномерное прямолинейное движение 51
СР-2. Прямолинейное движение с постоянным ускорением 52
СР-3. Свободное падение. Баллистическое движение 53
СР-4. Кинематика периодического движения 54
СР-5. Законы Ньютона 56
СР-6. Силы в механике 57
СР-7. Применение законов Ньютона 58
СР-8. Закон сохранения импульса 59
СР-9. Работа силы. Мощность 61
СР-10. Потенциальная и кинетическая энергия. Закон сохранения энергии 62
СР-11. Абсолютно неупругое и абсолютно упругое столкновение 63
СР-12. Движение тел в гравитационном поле 64
СР-13. Динамика свободных и вынужденных колебаний 66
СР-14. Релятивистская механика 67
СР-15. Молекулярная структура вещества 68
СР-16. Температура. Основное уравнение молекулярно-кинетической теории 69
СР-17. Уравнение Клапейрона-Менделеева. Изопроцессы 70
СР-18. Внутренняя энергия. Работа газа при изопроцессах 72
СР-19. Первый закон термодинамики 73
СР-20. Тепловые двигатели 74
СР-21. Испарение и конденсация. Насыщенный пар. Влажность воздуха 75
СР-22. Поверхностное натяжение. Смачивание, капиллярность 77
СР-23. Кристаллизация и плавление твердых тел. Механические свойства твердых тел 78
СР-24. Механические и звуковые волны 80
СР-25. Закон сохранения заряда. Закон Кулона 81
СР-26. Напряженность электростатического поля 83
СР-27. Работа сил электростатического поля. Потенциал 84
СР-28. Диэлектрики и проводники в электростатическом поле 86
СР-29. Электроемкость. Энергия электростатического поля 87
КОНТРОЛЬНЫЕ РАБОТЫ
КР-1. Прямолинейное движение 89
КР-2. Свободное падение тел. Баллистическое движение 93
КР-3. Кинематика периодического движения 97
КР-4. Законы Ньютона 101
КР-5. Применение законов Ньютона 105
КР-6. Закон сохранения импульса 109
КР-7. Закон сохранения энергии 113
КР-8- Молекулярно-кинетическая теория идеального газа 117
КР-9. Термодинамика 121
КР-10. Агрегатные состояния вещества 125
КР-11. Механические и звуковые волны 129
КР-12. Силы электромагнитного взаимодействия неподвижных зарядов 133
КР-13. Энергия электромагнитного взаимодействия неподвижных зарядов 137
ОТВЕТЫ
Тесты для самоконтроля 141
Самостоятельные работы 144
Контрольные работы 149
Список литературы 154.


Бесплатно скачать электронную книгу в удобном формате, смотреть и читать:
Скачать книгу Физика, 10 класс, Дидактические материалы, Марон А.Е., Марон Е.А., 2005 — fileskachat.com, быстрое и бесплатное скачивание.

  • Разноуровневые тестовые задания по физике, 10-11 класс, Федорова Н.Б., Ермаков Н.И., Кузнецова О.В., Борисова М.А., 2011

Тренировочные задания по физике марон. Дидактические по физике для девятиклассников и решебники к ним

Данное пособие включает тесты для самоконтроля, самостоятельные работы, разноуровневые контрольные работы.
Предлагаемые дидактические материалы составлены в полном соответствии со структурой и методологией учебников В. А. Касьянова «Физика. Базовый уровень. 10 класс» и «Физика. Углубленный уровень. 10 класс».

Примеры заданий:

ТС 1. Перемещение. Скорость.
Равномерное прямолинейное движение
Вариант 1
1. Двигаясь равномерно, велосипедист проезжает 40 м за 4 с. Какой путь он проедет при движении с той же скоростью за 20 с?
А. 30 м. Б. 50 м. В. 200 м.
2. На рисунке 1 приведен график движения мотоциклиста. Определите по графику путь, пройденный мотоциклистом в промежуток времени от 2 до 4 с.
А. 6м. Б. 2 м. В. 10 м.
3. На рисунке 2 представлены графики движения трех тел. Какой из этих графиков соответствует движению с большей скоростью?
А. 1. Б. 2. В. 3.
4. По графику движения, представленному на рисунке 3, определите скорость тела.
А. 1 м/с. Б. 3 м/с. В. 9 м/с.
5. Две автомашины движутся по дороге с постоянными скоростями 10 и 15 м/с. Начальное расстояние между машинами равно 1 км. Определите, за какое время вторая машина догонит первую.
А. 50 с. Б. 80 с. В. 200 с.

Предисловие.
ТЕСТЫ ДЛЯ САМОКОНТРОЛЯ
ТС-1. Перемещение. Скорость.
Равномерное прямолинейное движение.
ТС-2. Прямолинейное движение с постоянным ускорением
ТС-3. Свободное падение. Баллистическое движение.
ТС-4. Кинематика периодического движения.
ТС-5. Законы Ньютона.
ТС-6. Силы в механике.
ТС-7. Применение законов Ньютона.
ТС-8. Закон сохранения импульса.
ТС-9. Работа силы. Мощность.
ТС-10. Потенциальная и кинетическая энергия.
ТС-11. Закон сохранения механической энергии.
ТС-12. Движение тел в гравитационном поле.
ТС-13. Динамика свободных и вынужденных колебаний.
ТС-14. Релятивистская механика.
ТС-15. Молекулярная структура вещества.
ТС-16. Температура. Основное уравнение молекулярно-кинетической теории.
ТС-17. Уравнение Клапейрона-Менделеева. Изопроцессы.
ТС-18. Внутренняя энергия. Работа газа при изопроцессах. Первый закон термодинамики.
ТС-19. Тепловые двигатели.
ТС-20. Испарение и конденсация. Насыщенный пар. Влажность воздуха. Кипение жидкости.
ТС-21. Поверхностное натяжение. Смачивание, капиллярность.
ТС-22. Кристаллизация и плавление твердых тел.
ТС-23. Механические свойства твердых тел.
ТС-24. Механические и звуковые волны.
ТС-25. Закон сохранения заряда. Закон Кулона.
ТС-26. Напряженность электростатического поля.
ТС-27. Работа сил электростатического поля. Потенциал электростатического поля.
ТС-28. Диэлектрики и проводники в электростатическом поле.
ТС-29. Электроемкость уединенного проводника и конденсатора. Энергия электростатического поля.
САМОСТОЯТЕЛЬНЫЕ РАБОТЫ
СР-1. Равномерное прямолинейное движение.
СР-2. Прямолинейное движение с постоянным ускорением.
СР-3. Свободное падение. Баллистическое движение.
СР-4. Кинематика периодического движения.
СР-5. Законы Ньютона.
СР-6. Силы в механике.
СР-7. Применение законов Ньютона.
СР-8. Закон сохранения импульса.
СР-9. Работа силы. Мощность.
СР-9. Работа силы. Мощность.
СР-10. Потенциальная и кинетическая энергия. Закон сохранения энергии.
СР-11. Абсолютно неупругое и абсолютно упругое столкновение.
СР-12. Движение тел в гравитационном поле.
СР-13. Динамика свободных и вынужденных колебаний.
СР-14. Релятивистская механика.
СР-15. Молекулярная структура вещества.
СР-16. Температура. Основное уравнение молекулярно-кинетической теории.
СР-17. Уравнение Клапейрона-Менделеева. Изопроцессы.
СР-18. Внутренняя энергия. Работа газа при изопроцессах.
СР-19. Первый закон термодинамики.
СР-20. Тепловые двигатели.
СР-21. Испарение и конденсация. Насыщенный пар. Влажность воздуха.
СР-22. Поверхностное натяжение. Смачивание, капиллярность.
СР-23. Кристаллизация и плавление твердых тел. Механические свойства твердых тел.
СР-24. Механические и звуковые волны.
СР-25. Закон сохранения заряда. Закон Кулона.
СР-26. Напряженность электростатического поля.
СР-27. Работа сил электростатического поля. Потенциал.
СР-28. Диэлектрики и проводники в электростатическом поле.
СР-29. Электроемкость. Энергия электростатического поля
КОНТРОЛЬНЫЕ РАБОТЫ
КР-1. Прямолинейное движение.
КР-2. Свободное падение тел. Баллистическое движение.
КР-3. Кинематика периодического движения.
КР-4. Законы Ньютона.
КР-5. Применение законов Ньютона.
КР-6. Закон сохранения импульса.
КР-7. Закон сохранения энергии.
КР-8. Молекулярно-кинетическая теория идеального газа
КР-9. Термодинамика.
КР-10. Агрегатные состояния вещества.
КР-11. Механические и звуковые волны.
КР-12. Силы электромагнитного взаимодействия неподвижных зарядов.
КР-13. Энергия электромагнитного взаимодействия неподвижных зарядов.
ОТВЕТЫ
Тесты для самоконтроля.
Самостоятельные работы.
Контрольные работы.
Список литературы.


Бесплатно скачать электронную книгу в удобном формате, смотреть и читать:
Скачать книгу Физика, 10 класс, дидактические материалы к учебникам Касьянова В.А., Марон А.Е., 2014 — fileskachat.com, быстрое и бесплатное скачивание.

  • Физика, 10 класс, базовый уровень, учебник, Касьянов В.А., 2014

Знания по физике кому-то даются легко, а некоторым ребятам и вовсе не понятна суть многих явлений. К сожалению, учителя сейчас не могут уделять много времени подробным разъяснениям, поэтому школьникам в основном приходится самим постигать новую информацию. Но нет никаких гарантий, что они делают это правильно. Любые недочеты можно сразу же исправить, если использовать для занятий решебник к пособию «Физика 9 класс Дидактические материалы Марон Дрофа» .

Содержимое издания

Данный сборник может оказаться для школьников очень полезен, ведь в него вошли:

  • самостоятельные работы;
  • контрольные номера;
  • тесты.

«ГДЗ по физике 9 класс Марон» включает в себя самые полные и доскональные ответы по всем пунктам, поэтому ребятам не составит труда подготовиться к периодическим испытаниям.

Чем поможет решебник

Самостоятельное решение представленных номеров поможет ребятам понять, насколько хорошо они знают тематику, выявить свои слабые места и устранить их в кратчайшие сроки. Все данные, представленные в решебнике к пособию «Физика 9 класс Дидактические материалы Марон» проработаны авторами самым подробным образом, поэтому вниманию подростков предоставляется вполне доступная и понятная информация.

  • Опытнейшие педагоги-физики Марон А.Е и Марон Е.А. разработали замечательные Дидактические материалы, помогающие школьникам 9 класса успешно освоить нелегкий курс физики. Пособие содержит решения задач, задания для тренировки, тесты – контрольные и для самопроверки. Все работы представлены четырьмя вариантами.
  • Используя пособие, школьники улучшают результаты по трудному предмету, приобретают уверенность. Впереди — пугающая девятиклассников и родителей государственная аттестация, кроме прочных знаний требуется и психологическая устойчивость.
  • Некоторым школьникам невероятно тяжело дается любимый предмет Альберта Эйнштейна, хотя многие признают важность предмета для умственного развития, практической жизнедеятельности, формирования научного мировоззрения. Помощь таким ребятам окажет предлагаемое ГДЗ – здесь содержатся ответы и полные решения.
  • При разумном подходе школьник экономит силы и время, выстраивая самостоятельную работу оптимальным образом. Разобрав предлагаемое решение, ученик справляется потом и сам с подобными заданиями.
  • Бесценным помощником решебник становится родителям – контроль ДЗ проходит надёжно и быстро. Не стоит ослаблять родительский контроль девятиклассника, так ребенку легче получить качественное образование.
  • Дидактические по физике для девятиклассников и решебники к ним

  • Занимаясь регулярно с дидактическими материалами по физике для 9 класса, составленными Марон Е. А. и А. Е., девятиклассники в полном объеме освоят на практике такие разделы и тематики курса, как:
    — перемещение и путь;
    — движение — равномерное и прямолинейное, его относительность, равноускоренное движение;
    — основные законы Ньютона;
    — закон всемирного тяготения и свободное падение тел;
    — импульсы и законы сохранения энергии;
    — звук и механическое колебание волны;
    — электромагнитные поля;
    — строение атомного ядра и атома в целом.
    Изначально комплект материалов предназначался к базовому учебнику Перышкина А. В. по дисциплине. Но, учитывая разноплановость заданий, вскоре был признан экспертами как универсальное пособие, позволяющее использовать его в комплекте с различными программами и УМК по предмету. Для того, чтобы освоить все задания, представленные в сборнике, самостоятельно, специалисты рекомендуют применить решебник к нему. В этом случае, можно наглядно увидеть, как именно надо решать и записывать ответы на все предложенные в книге:
    — тренировочные упражнения;
    — тестовые материалы для самоконтроля;
    самостоятельные работы.
  • Занятия по ГДЗ можно организовать самостоятельно или привлечь в помощь репетиторов, педагогов-предметников, руководителей курсов и кружков по предмету. Особенно актуален четкий и грамотный план работы для тех, кто планирует участвовать в олимпиадах и конкурсах по дисциплине. Пособие может пригодиться и тем выпускникам, кто планирует сдавать физику в качестве предмета по выбору на ОГЭ. Также нередко его включают в число своих источников выпускники одиннадцатого класса, которые предпочли физику на ЕГЭ.
  • Начав занятия, следует придерживаться принципов:
    — планомерности и системности, ориентации на индивидуальные задачи, цели, пути их достижения, инструменты и базовый уровень знаний учащегося;
    — самоконтроля и регулярной самопроверки достигнутых результатов, выявлении и своевременной корректировки планов, устранении возникающих проблем;
    — грамотного планирования времени, которое будет тратиться на регулярную работу.
    В самом сборнике даны примеры решения типовых задач по физике для девятиклассников, а готовые домашние задания позволят в полной мере отследить и понять порядок и схемы решения всех представленных в пособии задач и упражнений, тестов.

Гдз по физике 10 дидактические материалы. Дидактические материалы по физике (10 класс)

Тесты для самоконтроля
Самостоятельные работы
Разноуровневые контрольные работы
УДК 373.167.1:53 ББК 22.3я72 М28

Марон, А. Е.

М28 Физика. 10 класс: дидактические материалы / А. Е. Ма­рон* Е. А. Марон. — 2-е изд., стереотип. — М.: Дрофа, 2005. — 156, с. : ил.

ШВК 5-7107-9105-9

Данное пособие включает тесты для самоконтроля, самостоятельные работы, разноуровневые контрольные работы.

Предлагаемые дидактические материалы составлены в полном соответствии со структурой и методологией учебника В. А. Касьянова «Физика. 10 класс*.

УДК 373.167.1:63 ББК 22.3я72

ISBN 5-7107-9105-9

О ООО «Дрофа*, 2004

Предисловие

Предлагаемые дидактические материалы входят в учебно-методическое обеспечение образовательных про­грамм по физике, рекомендованных Министерством об­разования Российской Федерации для средней школы, и составлены в полном соответствии со структурой и мето­дологией учебника В. А. Касьянова «Физика. 10 класс».

Пособие включает тесты для самоконтроля (ТС), само­стоятельные работы (СР) и контрольные работы (КР).

Комплект предусматривает организацию всех основ­ных этапов учебно-познавательной деятельности школь­ников: применение и актуализацию теоретических зна­ний, самоконтроль качества усвоения материала, выпол­нение самостоятельных и контрольных работ.

Тесты для самоконтроля с выбором ответа предназна­чены для проведения оперативного поурочного тематиче­ского контроля и самоконтроля знаний. В зависимости от конкретных условий (подготовка класса, организация разноуровневого обучения и т. д.) учитель может варьи­ровать набор тестовых заданий и определять время их выполнения.

Самостоятельные работы содержат 5 вариантов и рас­считаны примерно на 20 минут каждая.

Контрольные работы являются тематическими. Они рассчитаны на один урок и составлены в четырех вариан­тах. Каждый вариант содержит блоки задач разных уров­ней сложности, которые отделены в пособии друг от дру­га чертой. Первый и второй уровни сложности (I и И) соответствуют требованиям к уровню подготовки вы­пускников средней школы, третий уровень (111) преду­сматривает углубленное изучение физики. Самостоятель­ные и разноуровневые контрольные работы, тесты для самоконтроля, включенные в общую систему организа­ции активной учебно-познавательной деятельности уча­щихся, позволяют сформировать такие важные качества личности, как активность, самостоятельность, самодиаг­ностика и самооценка учебных достижений.

Всего в комплекте содержится более 1000 задач и за­даний, к большинству из которых приведены ответы.

3

Тесты для Самоконтроля
ТС-1. Перемещение. Скорость. Равномерное прямолинейное движение

Вариант 1


  1. Двигаясь равномерно, велосипедист проезжает 40 м за 4 с. Какой путь он проедет при движении с той же скоро­стью за 20 с?
А. 30 м. Б. 50 м. В. 200 м.

  1. На рисунке 1 приведен график движения мотоциклис­та. Определите по графику путь, пройденный мотоцик­листом в промежуток времени от 2 до 4 с.
А. 6 м. Б. 2 м. В. 10 м.

  1. На рисунке 2 представлены графики движения трех тел. Какой из этих графиков соответствует движению с большей скоростью?
А. 1 . Б. 2. В. 3.

  1. По графику движения, представленному на рисунке 3, определите скорость тела.
А. 1 м/с. Б. 3 м/с. В. 9 м/с.

  1. Две автомашины движутся по дороге с постоянными скоростями 10 и 15 м/с. Начальное расстояние между машинами равно 1 км. Определите, за какое время вто­рая машина догонит первую.
А. 50 с. Б. 80 с. В. 200 с.

Вариант 2


  1. Катер, двигаясь равномерно, проезжает 60 м за 2 с. Рассчитайте, какой путь он проедет за 10 с, двигаясь с той же скоростью.
А. 300 м. Б. 500 м. В. 100 м.

4


X, м,

х, м

1

X, м

12



/.

2

8

8

/

0

1 3

Ь, с

С

0

Рис. 4

Рис. 5



  1. Определите по графику движения (рис. 4) путь, прой­денный автомобилем в промежуток времени от 1 до 3 с.
Л. 8 м. Б. 4 м. В. 12 м.

  1. На рисунке 5 представлены три графика движения. Какой из этих графиков соответствует движению с мень­шей скоростью?
А. 1 . Б. 2. В. 3.


  1. По графику движения (рис. 6 ) определите скорость тела.
А. 8 м/с. Б. 4 м/с. В. 2 м/с.

  1. Колонна машин движется по шоссе со скоростью 10 м/с, растянувшись на расстояние 2 км. Из хвоста ко­лонны выезжает мотоциклист со скоростью 20 м/с и дви­жется к голове колонны. За какое время он достигнет го­ловы колонны?
А. 200 с. Б. 60 с. В. 40 с.

ТС-2. Прямолинейное движение с постоянным ускорением

Вариант 1


  1. Определите, какой из графиков (рис. 7) соответствует равнозамедленному движению тела.
А. 1.
Б. 2.

В. 3.

2. По графику зависимости скорос­ти от времени (рис. 8 ) определите ус­корение тела.

А. 0,5 м/с2. В. 4 м/с2.

Б. 2 м/с2.

с

5

Vf

t, с

t, с

Рис. 10


  1. Определите, на каком из графиков (рис. 9) представле­но движение тела, имеющего наименьшее ускорение.
А. 1. Б .2. В. 3.


  1. По графику зависимости скорости автомобиля от вре­мени (рис. 10 ) определите перемещение автомобиля за первые 3 с его движения.
А. 60 м. Б. 90 м. В. 30 м.

  1. Тело движется без начальной скорости с ускорением

  1. 5 м/с2. Определите путь, пройденный телом за первую секунду.
А. 0,25 м. Б. 1 м. В. 0,5 м.

Вариант 2


  1. Определите, какой из графиков (рис. 11) соответствует равноускоренному движению тела.
А. 1. Б .2. В. 3.

  1. По графику зависимости скорости от времени (рис. 12) определите ускорение тела.
А. 5 м/с2. Б. 1 м/с2. В. 2 м/с2.



  1. Определите, на каком из графиков (рис. 13) представ­лено движение тела, имеющего наибольшее ускорение.
А . 1. Б. 2. В. 3.


  1. По графику зависимости скорости мотоциклиста от времени (рис. 14) определите перемещение мотоциклиста за первые 2 с его движения.
А. 40 м. Б. 30 м. В. 20 м.

  1. После старта гоночный автомобиль достиг скорости 360 км/ч за 25 с. Определите расстояние, пройденное ав­томобилем за это время.
А. 1250 м. Б. 1400 м. В. 1500 м.

ТС-3. Свободное падение. Баллистическое движение1

Вариант 1


  1. Чему равна скорость свободно падающего тела через

  1. с после начала падения, если и0 = 0 ?
А. 20 м/с. Б. 10 м/с. В. 30 м/с.

  1. С какой высоты был сброшен предмет, если он упал на землю через 2 с?
А. 30 м. Б. 20 м. В. 10 м.

  1. Рассчитайте время свободного падения тела с высоты 20 м.
А. 1с. Б. 3 с. В. 2 с.

  1. Тело брошено вертикально вверх со скоростью 30 м/с. Какова максимальная высота подъема тела?
А. 45 м. Б. 50 м. В. 90 м.
1 При решении задач принять Ј = 10 м/с2

7



  1. Мяч бросают с крыши, находящейся на высоте 20 м от
    поверхности земли. Его начальная скорость равна 25 м/с
    и направлена горизонтально. Чему равна дальность поле-
    та мяча по горизонтали?
А. 50 м. Б. 100 м. В. 75 м.

Вариант 2


  1. Определите скорость свободно падающего тела через

  1. с после начала падения, если и0 —
0 .

А. 10 м/с. Б. 30 м/с. В. 20 м/с.


  1. Какова глубина ущелья, если упавший в него камень
    коснулся дна через 4 с?
А. 80 м. Б. 100 м. В. 150 м.

  1. Мяч упал на землю с высоты 80 м. Определите, сколь-
    ко времени мяч находился в полете.
А. 2 с. Б. 1 с. В. 4 с.

  1. Стрела выпущена из лука вертикально вверх со скоро-
    стью 20 м/с. Рассчитайте максимальную высоту подъема
    стрелы.
А. 10 м. Б. 20 м. В. 30 м.

  1. Самолет летит горизонтально на высоте 8 км со скоро-
    стью 1800 км/ч. За сколько километров до цели летчик
    должен сбросить бомбу, чтобы поразить цель?
А. 40 км. Б. 20 км. В. 10 км.

ТС-4. Кинематика периодического движения

Вариант 1


  1. Тело движется равномерно по окружности против ча-
    совой стрелки (рис. 15). Как направлен вектор ускорения
при таком движении?

А.

1. В. 3.

Б.

2.

2. Тело движется по окружности радиусом


  1. м со скоростью Юл м/с. Определите пери-
    од вращения тела.
А. 0,8 с. В. 2 с.

Б. 1 с. Рис. 15

8


  1. Мотоциклист совершает поворот по круговой траекто­рии радиусом 50 м с постоянной по модулю скоростью 10 м/с. Каково ускорение мотоциклиста?
А. 1 м/с2. Б. 3 м/с2. В. 2 м/с2.

  1. Частица совершает гармонические колебания по зако­ну х =
10 cos nt/10 см. Определите координату частицы в момент времени t = 10 с.

А. 10 см. Б. -10 см. В. 0.


  1. t = 10 с.
А. 1 м/с. Б. 0. В. 2 м/с.

Вариант 2


  1. Тело движется равномерно по окруж-
    ности по часовой стрелке (рис. 16). Как
    направлен вектор ускорения при таком
    движении?
А. 1 . В. 3.

Б.

2.

  1. Какова частота вращения тела, движущегося по ок­ружности радиусом 5 м со скоростью 5 п м/с?
А. 2 Гц. Б. 0,5 Гц. В. 4 Гц.

  1. Трамвайный вагон движется на повороте по закругле­нию радиусом 40 м. Рассчитайте скорость трамвая, если центростремительное ускорение равно 0,4 м/с2.
А. 2 м/с. Б. 1 м/с. В. 4 м/с.

  1. Тело совершает гармонические колебания по закону х = 5 cos nt/6 см. Определите координату тела в момент времени Ј = 2 с.
А. 2,5 см. Б. 2 см. В. 0,4 см.

  1. По условию предыдущей задачи определите скорость частицы в момент времени t = 6 с.
А. 0. Б. 1 м/с. В. 0,5 м/с.

Данное пособие включает тесты для самоконтроля, самостоятельные работы, разноуровневые контрольные работы.
Предлагаемые дидактические материалы составлены в полном соответствии со структурой и методологией учебников В. А. Касьянова «Физика. Базовый уровень. 10 класс» и «Физика. Углубленный уровень. 10 класс».

Примеры заданий:

ТС 1. Перемещение. Скорость.
Равномерное прямолинейное движение
Вариант 1
1. Двигаясь равномерно, велосипедист проезжает 40 м за 4 с. Какой путь он проедет при движении с той же скоростью за 20 с?
А. 30 м. Б. 50 м. В. 200 м.
2. На рисунке 1 приведен график движения мотоциклиста. Определите по графику путь, пройденный мотоциклистом в промежуток времени от 2 до 4 с.
А. 6м. Б. 2 м. В. 10 м.
3. На рисунке 2 представлены графики движения трех тел. Какой из этих графиков соответствует движению с большей скоростью?
А. 1. Б. 2. В. 3.
4. По графику движения, представленному на рисунке 3, определите скорость тела.
А. 1 м/с. Б. 3 м/с. В. 9 м/с.
5. Две автомашины движутся по дороге с постоянными скоростями 10 и 15 м/с. Начальное расстояние между машинами равно 1 км. Определите, за какое время вторая машина догонит первую.
А. 50 с. Б. 80 с. В. 200 с.

Предисловие.
ТЕСТЫ ДЛЯ САМОКОНТРОЛЯ
ТС-1. Перемещение. Скорость.
Равномерное прямолинейное движение.
ТС-2. Прямолинейное движение с постоянным ускорением
ТС-3. Свободное падение. Баллистическое движение.
ТС-4. Кинематика периодического движения.
ТС-5. Законы Ньютона.
ТС-6. Силы в механике.
ТС-7. Применение законов Ньютона.
ТС-8. Закон сохранения импульса.
ТС-9. Работа силы. Мощность.
ТС-10. Потенциальная и кинетическая энергия.
ТС-11. Закон сохранения механической энергии.
ТС-12. Движение тел в гравитационном поле.
ТС-13. Динамика свободных и вынужденных колебаний.
ТС-14. Релятивистская механика.
ТС-15. Молекулярная структура вещества.
ТС-16. Температура. Основное уравнение молекулярно-кинетической теории.
ТС-17. Уравнение Клапейрона-Менделеева. Изопроцессы.
ТС-18. Внутренняя энергия. Работа газа при изопроцессах. Первый закон термодинамики.
ТС-19. Тепловые двигатели.
ТС-20. Испарение и конденсация. Насыщенный пар. Влажность воздуха. Кипение жидкости.
ТС-21. Поверхностное натяжение. Смачивание, капиллярность.
ТС-22. Кристаллизация и плавление твердых тел.
ТС-23. Механические свойства твердых тел.
ТС-24. Механические и звуковые волны.
ТС-25. Закон сохранения заряда. Закон Кулона.
ТС-26. Напряженность электростатического поля.
ТС-27. Работа сил электростатического поля. Потенциал электростатического поля.
ТС-28. Диэлектрики и проводники в электростатическом поле.
ТС-29. Электроемкость уединенного проводника и конденсатора. Энергия электростатического поля.
САМОСТОЯТЕЛЬНЫЕ РАБОТЫ
СР-1. Равномерное прямолинейное движение.
СР-2. Прямолинейное движение с постоянным ускорением.
СР-3. Свободное падение. Баллистическое движение.
СР-4. Кинематика периодического движения.
СР-5. Законы Ньютона.
СР-6. Силы в механике.
СР-7. Применение законов Ньютона.
СР-8. Закон сохранения импульса.
СР-9. Работа силы. Мощность.
СР-9. Работа силы. Мощность.
СР-10. Потенциальная и кинетическая энергия. Закон сохранения энергии.
СР-11. Абсолютно неупругое и абсолютно упругое столкновение.
СР-12. Движение тел в гравитационном поле.
СР-13. Динамика свободных и вынужденных колебаний.
СР-14. Релятивистская механика.
СР-15. Молекулярная структура вещества.
СР-16. Температура. Основное уравнение молекулярно-кинетической теории.
СР-17. Уравнение Клапейрона-Менделеева. Изопроцессы.
СР-18. Внутренняя энергия. Работа газа при изопроцессах.
СР-19. Первый закон термодинамики.
СР-20. Тепловые двигатели.
СР-21. Испарение и конденсация. Насыщенный пар. Влажность воздуха.
СР-22. Поверхностное натяжение. Смачивание, капиллярность.
СР-23. Кристаллизация и плавление твердых тел. Механические свойства твердых тел.
СР-24. Механические и звуковые волны.
СР-25. Закон сохранения заряда. Закон Кулона.
СР-26. Напряженность электростатического поля.
СР-27. Работа сил электростатического поля. Потенциал.
СР-28. Диэлектрики и проводники в электростатическом поле.
СР-29. Электроемкость. Энергия электростатического поля
КОНТРОЛЬНЫЕ РАБОТЫ
КР-1. Прямолинейное движение.
КР-2. Свободное падение тел. Баллистическое движение.
КР-3. Кинематика периодического движения.
КР-4. Законы Ньютона.
КР-5. Применение законов Ньютона.
КР-6. Закон сохранения импульса.
КР-7. Закон сохранения энергии.
КР-8. Молекулярно-кинетическая теория идеального газа
КР-9. Термодинамика.
КР-10. Агрегатные состояния вещества.
КР-11. Механические и звуковые волны.
КР-12. Силы электромагнитного взаимодействия неподвижных зарядов.
КР-13. Энергия электромагнитного взаимодействия неподвижных зарядов.
ОТВЕТЫ
Тесты для самоконтроля.
Самостоятельные работы.
Контрольные работы.
Список литературы.

Бесплатно скачать электронную книгу в удобном формате, смотреть и читать:


Скачать книгу Физика, 10 класс, дидактические материалы к учебникам Касьянова В.А., Марон А.Е., 2014 — fileskachat.com, быстрое и бесплатное скачивание.
  • Физика, 10 класс, базовый уровень, учебник, Касьянов В.А., 2014

Учебнометодическое пособие
2е изд., стереотип.
М.: Дрофа, 2005. 156 стр.
Сканированные страницы, текстовый слой оутвует

Оглавление
Предисловие
Перемещение. Скорость.Равномерное прямолинейное движение

Свободное падение. Баллистическое движение.

Законы Ньютона
Силы в механике
Применение законов Ньютона
Закон сохранения импульса
Работа силы. Мощность
Потенциальная и кинетическая энергия
Закон сохранения механической энергии

Релятивиская механика

Температура. Основное уравнение молекулярнокинетической теории

Внутренняя энергия. Работа газа при изопроцессах.Первый закон термодинамики.
Тепловые двигатели
Испарение и конденсация. Насыщенный пар. Влажность воздуха. Кипение жидкости

Кристаллизация и плавление твердых тел
Механические свойства твердых тел
Механические и звуковые волны

Работа сил электростатического поля.Потенциал электростатического поля

Электроемкость уединенного проводника и конденсатора. Энергия электростатического поля

Самостоятельные работы
Равномерное прямолинейное движение
Прямолинейное движение с постоянным ускорением
Свободное падение. Баллистическое движение
Кинематика периодического движения
Законы Ньютона
Силы в механике
Применение законов Ньютона
Закон сохранения импульса
Работа силы. Мощность
Потенциальная и кинетическая энергия. Закон сохранения энергии
Абсолютно неупругое и абсолютно упругое столкновение
Движение тел в гравитационном поле
Динамика свободных и вынужденных колебаний
Релятивиская механика
Молекулярная структура вещества
Температура. Основное уравнение. молекулярнокинетической теории
Уравнение Клапейрона-Менделеева. Изопроцессы
Внутренняя энергия. Работа газа при изопроцессах
Первый закон термодинамики
Тепловые двигатели
Испарение и конденсация. Насыщенный пар. Влажность воздуха.
Поверхностное натяжение. Смачивание, капиллярность
Кристаллизация и плавление твердых тел. Механические свойства твердых тел.
Механические и звуковые волны
Закон сохранения заряда. Закон Кулона
Напряженность электростатического поля
Работа сил электростатического поля. Потенциал
Диэлектрики и проводники в электростатическом поле
Электроемкость. Энергия электростатического поля

Контрольные работы
Прямолинейное движение
Свободное падение тел. Баллистическое движение
Кинематика периодического движения
Законы Ньютона
Применение законов Ньютона
Закон сохранения импульса
Закон сохранения энергии
Молекулярнокинетическая теория идеального газа
Термодинамика
Агрегатные состояния вещества
Механические и звуковые волны
Силы электромагнитного взаимодействия неподвижных зарядов
Энергия электромагнитного взаимодействия неподвижных зарядов

Ответы
Тесты для самоконтроля
Самостоятельные работы
Контрольные работы
Список литературы

Входная контрольная работа по физике — 10 класс

Вариант №1

1) Космонавт проводит ремонт космического корабля.

2) Космонавт в космическом корабле обращается вокруг Земли.

2.По какой формуле определяется ускорение тела?

3. Какой формулой выражается второй закон Ньютона?

1 = -F 2 ; 4) F = m a.

4. Какая волна называется продольной?

5. По какой формуле определяется импульс тела?

6. При равноускоренном движении скорость тела за 5с увеличилась на 4 м/с. Чему равно ускорение тела? (в м / с 2)

7. Определить скорость звука в воде, если источник, колеблющийся с периодом 0,002 с, возбуждает в воде волны длиной 2,9 м.

1) 1483 м/с; 2) 1450 м/с; 3) 1385 м/с; 4) 1567 м/с.

8. Чему равна сила тяжести, действующая на тело массой 1,2 т?

1) 1200 Н; 2) 12000 Н; 3) 120 Н; 4) 0, 12 Н.

9. Частота колебаний железнодорожного моста длиной 100м равна 2 Гц. Определить период этих колебаний.

1) 2 с; 2) 4 с; 3) 0,5 с; 4) 0, 2 с.

10. Велосипедист равномерно движется в течение 5 с со скоростью 2 м/с. Какое расстояние он пройдет за данное время?

1) 1 м; 2) 4 м; 3) 2,5 м; 4) 10 м.

Вариант №2.

1. Какую физическую величину определяет водитель автомобиля по счетчику спидометра?

1) перемещение;

2) пройденный путь;

3) траекторию.

2.По какой формуле определяется мгновенная скорость тела?

1) V = V 0 + at ; 2) a = V – V 0 / 2 ; 3) a = V – V 0 / t ; 4) V = V 0 – at.

3. Какой формулой выражается третий закон Ньютона?

1) F = m V ; 2) F = m g ; 3) F 1 = -F 2 ; 4) F = m a.

4. Какая волна называется поперечной?

1) волна, в которой колебания распространяются вдоль направления распространения волны;

2) волна, в которой колебания распространяются перпендикулярно направлению распространения волны;

3)волна, колебания в которой несвязанны с направлением распространения волны.

5. По какой формуле определяется импульс силы?

1) F t ; 2) E t ; 3) mV ; 4) F V .

6. При равноускоренном движении скорость тела за 2с увеличилась на 1,6 м/с. Чему равно ускорение тела? (в м / с 2)

1) 5 ; 2) 8 ; 3) 0,5 ; 4) 0, 8.

7. Определить силу, под действием которой велосипедист скатывается с горки с ускорением, равным 0, 8 м/с 2 , если масса велосипедиста вместе с велосипедом

равна 50 кг?

1) 50 Н; 2) 40 Н; 3) 60 Н; 4) 80 Н.

8. Определить длину звуковой волны частотой 725 Гц в стекле, если скорость распространения звука в стекле равна 5500 м/с.

1) 7, 59 м; 2) 14 м; 3) 759 м; 4) 1,4 м.

9. На тело, лежащее на столе действует сила тяжести равная 40 Н. Чему равна масса этого тела?

1) 400 кг; 2) 40 кг; 3) 4 кг; 4) 0, 4 кг

10. Чему равен импульс, движущегося со скоростью 2 м/с тела, если его масса равна 200г?

1) 0, 4 кг*м/с; 2) 400 кг*м/с; 3) 4 кг*м/с; 4) 0, 04 кг*м/с.

Код входной к/р 10 класс.

Контрольная работа №1 – 10 класс, по теме «Кинематика»

Вариант №1

1.Что называется механическим движением?

2.Написать формулу мгновенной скорости.

3. Что называется периодом вращения?

4 . Во время половодья скорость течения сибирской реки Тунгуски местами достигает значения υ = 30 км/ч. С какой по модулю скоростью υ будет двигаться относительно берегов быстроходный катер, пересекающий реку по кратчайшему пути? Скорость катера в стоячей воде υ 0 = 34 км/ч.

5. Из пункта A 1 = 15 км/ч и υ 2 t 0 t l от пункта A

6. Стоя над рекой на краю высокого обрыва высотой H = 99 м, мальчик бросил камень со скоростью υ = 10 м/с под углом α = 45° к горизонту. Пренебрегая сопротивлением воздуха, определите дальность L полета камня ( g = 10 м/с 2 ).

Вариант №2

1. Какое движение называется равномерным?

2. Написать формулу скорости при свободном падении.

3.Какое движение называется поступательным?

4. Поезд длиной l 1 = 225 м, движущийся с постоянной скоростью, проходит мимо телеграфного столба за время t 0 = 15 с. С какой скоростью движется поезд?

5. При торможении на прямолинейном участке дороги скорость автомобиля равномерно уменьшается от υ 1 = 20 м/с до υ 2 = 10 м/с за время t = 5 с. Определите модуль ускорения автомобиля.

6 . Какую скорость должен иметь искусственный спутник Земли при движении по круговой орбите на высоте h g = 9,23 м/с 2 . Радиус Земли R З = 6,4·10 6 м.

Вариант №3.

1. Что входит в систему отсчета?

2. Написать формулу центростремительного ускорения.

3.Что называется частотой вращения?

4. Сколько времени t 1 пройдет от момента вхождения поезда в туннель длиной L = 450 м до выхода из него последнего вагона? Если длина поезда 300 м, а его скорость 72 км/ч

5. Зависимость координат тела на плоскости XY от времени t (с) задана уравнениями

x = 0, y = t 2 + 4 t + 3 (м).

Определите начальную скорость и ускорение тела.

6. Согласно планетарной модели атома, созданной Резерфордом, электрон в атоме водорода движется по круговой орбите вокруг ядра с постоянной скоростью. Определите центростремительное ускорение электрона, если радиус орбиты R = 0,5·10 –10 м, а скорость электрона на этой орбите υ = 2,2·10 6 м/с.

Вариант №4.

1. Какое движение называется равнопеременным?

2.Написать формулу перемещения при равномерном движении.

3.Что называется материальной точкой?

4 .За какое время t 2 пройдет мимо машиниста этого поезда весь состав встречного поезда длиной l 2 = 300 м, движущегося со скоростью υ 2 = 36 км/ч? Если скорость самого поезда 56 км/ч.

5. При испытаниях автомобиля на скоростном прямолинейном участке пути он двигался сначала с ускорением a = 5 м/с 2 , затем равномерно и, наконец, замедляясь с тем же по модулю ускорением, остановился. Полное время движения автомобиля t = 37,5 с, средняя скорость за это время υ ср = 108 км/час. Начальная скорость автомобиля равнялась нулю. Определите, какое время автомобиль двигался равномерно.

6. Какую скорость должен иметь искусственный спутник Земли при движении по круговой орбите на высоте h = 200 км над поверхностью Земли? Ускорение свободного падения на этой высоте g = 9,23 м/с 2 . Радиус Земли R З = 6,4·10 6 м.

Вариант №5.

1.Какое движение называется неравномерным?

2.Написать формулу линейной скорости тела при движении по окружности.

3.Что называется перемещением.

4. Стрелок в тире длиной L = 100 м услышал звук от пули, попавшей в мишень, через время t = 0,45 с после выстрела. Принимая скорость звука равной υ зв = 330 м/с и пренебрегая искривлением траектории пули, определите ее скорость.

5. Свободно падающее без начальной скорости тело за последнюю секунду падения прошло 2/3 своего пути. Определите полное время падения t и весь путь H , пройденный телом (g = 10 м/с 2 ).

6. Из пункта A в одном направлении отправились два велосипедиста со скоростями υ 1 = 15 км/ч и υ 2 = 20 км/ч. Второй велосипедист выехал на t 0 = 1 ч позднее первого. Через какое время t после своего отправления и на каком расстоянии l от пункта A второй велосипедист догонит первого?

Тестовые задания по физике для 9 класса | Тест по физике (9 класс) по теме:

Контрольная работа по теме «Динамика» 9 класс

Вариант 1

1. Полосовой магнит массой М поднесли к проводнику массой m, по которому течет ток. Сравните силу действия магнита на проводник F1 с силой действия проводника на магнит F2.

       2.Как будет двигаться тело массой 6 кг под действием постоянной силы 12 Н?

       1) равномерно со скоростью 2 м/с;                2) равномерно со скоростью 0,5 м/с;

       3) равноускоренно с ускорением 0,5 м/с2 ;   4) равноускоренно с ускорением 2 м/с2   

     3. В инерциальной системе отсчета сила F сообщает телу массой m ускорение а. Как

       изменится ускорение тела, если массу тела и действующую на него силу уменьшить в

       2 раза?

      1) увеличится в 4 раза;                  2) уменьшится в 2 раза;

      3) не изменится;                              4) увеличится в 2 раза.                              

     4. Метеорит пролетает около Земли за пределами атмосферы. В тот момент, когда    

      вектор силы гравитационного притяжения Земли перпендикулярен вектору скорости    

      метеорита, вектор ускорения метеорита направлен

    1) параллельно вектору скорости;          2) по направлению вектора силы;

    3) по направлению вектора скорости;    4) по направлению суммы векторов силы      

                                                                              и   скорости.

                                                                                                         

      5. Две силы 3 Н и 4 Н приложены к одной точке тела, угол между векторами сил равен    

     90°. Модуль равнодействующей сил равен

         1) 1 Н          2) 5 Н             3) 7 Н             4) 25 Н                        

     6. Скорость движущегося вдоль оси ОХ тела массой 1,5 кг

    изменяется со временем в соответствии с графиком (см. рисунок).  

   Определите силу, действующую на тело

        1) 1,5 Н    2) 6 Н    3) 30Н        4) 60Н                              

7. Автомобиль движется со скоростью V1 = 90 км/ч, а мотоцикл со скоростью V2 = 180 км/ч. Масса мотоцикла m = 500 кг. Отношение импульса автомобиля к импульсу мотоцикла равно 1,5. Масса автомобиля равна

1) 1500 кг             2) 3000 кг                    3) 4000 кг            4) 8000 кг                  

8. Человек массой 50 кг прыгает из неподвижной лодки массой 100 кг на берег с горизонтальной скоростью 3 м/с относительно лодки. Если сопротивление воды движению лодки пренебрежимо мало, то лодка после прыжка человека движется относительно Земли со скоростью

1) 3 м/с                  2) 2 м/с                      3) 1,5 м/с             4) 1 м/с                        

9. Сила тяготения между двумя телами уменьшится в 2 раза, если массу каждого из тел

1) увеличить в  раз  2) уменьшить в  раз     3) увеличить в 2 раза  

4) уменьшить в 2 раза

10. Бруску массой m, лежащему на гладкой горизонтальной поверхности, сообщают горизонтальную скорость V, с которой он, двигаясь без трения, проходит точку 1. Когда брусок сместится на расстояние S относительно точки 1, его полная механическая энергия

1) увеличится на величину mgh              2) увеличится на величину/2

3) уменьшится на величину mgh              4) не изменится                                          

11. Как изменится потенциальная энергия тела, если при неизменной массе тела высоту над землей увеличить в 2 раза?

1) увеличится в 4 раза                2) увеличится в 2 раза

3) уменьшится  в 2 раза              4) не изменится                                          

12. Как изменится кинетическая энергия тела, если при неизменной массе тела его скорость увеличить в 2 раза?

1) увеличится в 4 раза                2) увеличится в 2 раза

3) уменьшится  в 2 раза              4) не изменится                                          

В1. Тело падает в воздухе с ускорением 8 м/с2, величина силы сопротивления воздуха постоянна и равна 10 Н. Найти массу тела. Чертеж обязателен.

В2. Мальчик массой 60 кг догоняет движущиеся в том же направлении сани массой 10 кг и запрыгивает на них. Перед прыжком скорость мальчика равна 2,6 м/с, скорость саней 2 м/с. Чему равна начальная скорость их совместного движения? Чертеж обязателен.

С1. Груз массой 1 кг вращают в вертикальной плоскости на веревке длиной 1 м. Чему равна сила натяжения веревки при прохождении телом верхней точки, если скорость тела в ней 1 м/с? Чертеж обязателен.

С2. Автобус массой 18 т трогается с места с ускорением 0,5 . Найти силу тяги, если сила сопротивления движению 9кН. Чертеж обязателен.

Контрольная работа по теме «Динамика» 9 класс

Вариант 2

1. Ядро атома массой М притягивает электрон массой m. Сравните силу действия ядра на электрон F1  с силой действия электрона на ядро F2.

2. Как будет двигаться тело массой 4 кг под действием постоянной силы 12 Н?

1) равномерно со скоростью 3 м/с.              2) равномерно со скоростью 0,3 м/с.

3) равноускоренно с ускорением 3 м/с2.     4) равноускоренно с ускорением 0,3 м/с2.    

3. В инерциальной системе отсчета сила F сообщает телу массой m ускорение a. Как надо изменить массу тела, чтобы вдвое большая сила сообщала ему в 4 раза меньшее ускорение?

1) увеличить в 2 раза                                           2) увеличить в 4 раза

3) увеличить в 8 раз                                            4) уменьшить в 2 раза                

4. Космический корабль улетает от Земли. Как направлен вектор ускорения корабля в тот момент, когда вектор силы гравитационного притяжения Земли направлен под углом 120° к вектору скорости корабля? Действие остальных тел на корабль пренебрежимо мало.

1) По направлению вектора скорости         2) По направлению вектора силы

3) Противоположно вектору скорости

4) По направлению суммы векторов силы и скорости            

5. На рисунке представлены три вектора сил, приложенных к одной точке и лежащих в одной плоскости. Модуль вектора силы F1 равен 5 Н. Модуль равнодействующей векторов F1, F2 и F3 равен

1) 11 Н       2) 7 Н           3) 5 Н             4) 0 Н      

6.,м/с                                             Скорость движущегося вдоль оси ОХ тела массой 2 кг

       7 —                                           изменяется со временем в соответствии с графиком

       6 —                                            Определите силу, действующую на тело

        5 —                              

        4 —                                               1. 0,5Н               2. 1Н           3. 2Н           4. 8Н        

       3 —                                      

       2 —                              

       1 —                                    

       0                                      

               1     2    3    4     5    6           t,с      

7. Масса грузовика m1=5000 кг, масса легкового автомобиля m2= 1000 кг. Грузовик движется со скоростью V= 72 км/ч. Отношение импульса грузовика к импульсу автомобиля равно 2,5. Скорость легкового автомобиля равна

1) 72 км/             2) 90 км/ч                     3) 108 км/ч             4) 144 км/ч                    

8. Человек массой m прыгает с горизонтальной скоростью V с берега в неподвижную лодку массой М. Каким суммарным импульсом обладают лодка с человеком, если сопротивление воды движению лодки пренебрежимо мало?

1) 0                      2) mV                            3) (m + M)V          4) mMV/(m + M)                  

9. Как измениться сила притяжения спутника к планете, если расстояние от центра спутника до центра планеты уменьшится в 2 раза?

1) увеличивается в 2 раза        3) уменьшается в 2 раза  

2) увеличивается в 4 раза          4) уменьшается в 4 раза        

10. Потенциальная энергия характеризует

1. движение тела.               2. взаимодействие тел или частей тела.                

3. плотность тела.                4. инертность тела.

11. Как изменится потенциальная энергия тела, если при неизменной массе тела высоту над землей уменьшить в 3 раза

1) увеличится в 3 раза                2) увеличится в 9 раза

3) уменьшится  в 3 раза              4) не изменится                                          

12. Как изменится кинетическая энергия тела, если при неизменной массе тела его скорость уменьшить в 2 раза?

1) увеличится в 4 раза                2) уменьшится в 4 раза

3) уменьшится  в 2 раза              4) не изменится                                          

В1. Лифт движется вверх с ускорением 2 м/с2. Чему равен вес находящегося в лифте человека массой 75 кг? Чертеж обязателен.

В2. Мальчик массой 60 кг догоняет движущиеся в том же направлении сани массой 10 кг и запрыгивает на них. Перед прыжком скорость мальчика равна 2,6 м/с, скорость саней 2 м/с. Чему равна начальная скорость их совместного движения? Чертеж обязателен.

С1. Тело массой 10 кг движется в горизонтальном направлении по поверхности под    

 действием силы 50 Н. С каким ускорением движется тело, если на него действует сила трения равная 40 Н? Чертеж обязателен.

С2. Масса автомобиля равна 1 т, а скорость его движения – 20 м/с. Чему равен радиус кривизны выпуклого моста, если вес автомобиля в верхней точке выпуклого моста равен 8 кН? Чертеж обязателен.

Контрольная работа по теме «Динамика» 9 класс

Вариант 3

1. Земля массой М притягивает находящееся на ее поверхности тело  массой m. Сравните силу действия Земли на это тело F1 с силой действия тела на Землю F2.

2. Как будет двигаться тело массой 10 кг под действием постоянной силы 20 Н?

1) равноускоренно с ускорением 0,5 м/с2         2) равноускоренно с ускорением 2 м/с2

3) равномерно со скоростью 2 м/с                      4) равномерно со скоростью 0,5 м/с        

3. В инерциальной системе отсчета сила F сообщает телу массой m ускорение a. Как надо изменить массу тела, чтобы втрое большая сила сообщала ему в 2 раза меньшее ускорение?

1) увеличить в 2 раза                                            2) увеличить в 6 раз

3) увеличить в 3 раза                                            4) уменьшить в 3 раза                    

4. Самолет выполняет фигуру высшего пилотажа «мертвая петля». Как направлен вектор ускорения самолета в тот момент времени когда вектор равнодействующей всех сил направлен вертикально вверх к центру окружности, а вектор скорости самолета направлен горизонтально?

1) Вертикально вверх                                         2) По направлению вектора скорости

3) Противоположно вектору скорости             4) Вертикально вниз                                  

5. Тело подвешено на двух нитях и находится в равновесии. Угол

между нитями равен 90°, а силы натяжения нитей равны 3 Н и 4 Н. Вес тела равен

1) 1 Н      2) 5 Н        3) 7 Н         4) 25 Н                            

6.        ,м/с                                     Скорость движущегося вдоль оси ОХ тела массой 10 кг

                     7 —                                 изменяется со временем в соответствии с графиком

                     6 —                                 (см. рисунок). Определите силу, действующую на тело

                     5 —                              

                     4 —                                            1. 4Н      2. 5Н     3. 10Н    4. 20Н      

                     3 —                                      

                     2 —                              

                     1 —                                  

                     0                                      

                             1     2    3   4     5    6                   t,с      

7. Масса мотоцикла m1= 500 кг, масса автомобиля m2= 1000 кг. Автомобиль движется со скоростью V= 108 км/ч. Отношение импульса автомобиля к импульсу мотоцикла равно 1,5. Скорость мотоцикла равна

1) 72 км/ч           2) 90 км/ч                  3) 108 км/ч                  4) 144 км/ч              

8. Вагон массой m, движущийся со скоростью V, сталкивается с неподвижным вагоном массой 2m. Каким суммарным импульсом обладают два вагона после столкновения? Взаимодействие вагонов с другими телами пренебрежимо мало.

1) 0                     2) mv/3                       3) mv/2                      4) mv                    

9. При увеличении расстояния между двумя шариками в 3 раза сила гравитационного притяжения одного шара к другому:

1) увеличивается в 3 раза     3) уменьшается в 3 раза

2) увеличивается в 9 раз       4) уменьшается в 9 раз                          

10. Мяч массой m бросают вертикально вверх. Если пренебречь сопротивлением воздуха, то при подъеме мяча на высоту h относительно первоначального положения, его полная механическая энергия

1) увеличится на величину mgh      2) уменьшится на величину mgh                

3) не изменится                 4) будет неизвестна, так как не задана начальная скорость

11. Как изменится потенциальная энергия тела, если при неизменной высоте над землей массу тела увеличить в 2 раза

1) увеличится в 4 раза                2) увеличится в 2 раза

3) уменьшится  в 2 раза              4) не изменится                                          

12. Какова масса тела, движущегося со скоростью 36км/ч, если его кинетическая энергия равна50кДж?

1) 500кг             2) 50кг            3) 1000кг          4) 360кг    

В1. Лифт движется вниз с ускорением 1 м/с2. Чему равен вес человека массой 80 кг? Чертеж обязателен.

В2. Вагон массой 20 т движется со скоростью 1,5 м/с и наталкивается на неподвижную платформу массой 10 т. С какой скоростью они станут двигаться в сцепке? Чертеж обязателен.

С1. С каким ускорением тело массой 70 кг погружается в воду, если выталкивающая сила,

действующая на него равна 500 Н, а сила сопротивления воды 130 Н? Чертеж обязателен.

С2. С какой скоростью должен двигаться мотоциклист по выпуклому участку дороги, имеющему радиус кривизны 40 м, чтобы в верхней точке выпуклости сила давления на дорогу была равна нулю? Чертеж обязателен.

Контрольная работа по теме «Динамика» 9 класс

Вариант 4

1. Брусок массой m лежит на доске массой М. Сравните силу действия доски на брусок Fl с силой действия бруска на доску F2.

2. Как будет двигаться тело массой 2 кг под действием постоянной силы 10 Н?

1) равномерно со скоростью 5 м/с                 2) равномерно со скоростью 0,2 м/с

3) равноускоренно с ускорением 0,2 м/с2    4) равноускоренно с ускорением 5 м/с2                                              

3. В инерциальной системе отсчета сила F сообщает телу массой m ускорение а.

Как изменится ускорение тела, если массу тела в 2 раза уменьшить, а действующую на

него силу вдвое увеличить?

1) увеличится в 4 раза                                   2) уменьшится в 2 раза

3) уменьшится в 4 раза                                  4) увеличится в 2 раза.                  

4. Самолет летит по окружности в горизонтальной плоскости с постоянной по модулю скоростью. Вектор ускорения самолета направлен

1) вертикально вверх;            2) к центру окружности ;

3) от центра окружности;       4)вертикально вниз.                                              

5. На рисунке представлены три вектора сил, лежащих в одной плоскости и приложенных к одной точке. Масштаб рисунка таков, что сторона одного квадрата сетки соответствует модулю силы 1 Н. Определите модуль вектора равнодействующей трех векторов сил.

1) 1Н                2) 5Н                3) 10Н                 4) 7Н                  

6.         ,м/с                                      Скорость движущегося вдоль оси ОХ тела массой 5 кг

                     7 —                                    изменяется со временем в соответствии с графиком

                     6 —                                   (см. рисунок). Определите силу, действующую на тело

                     5 —                                        

                     4 —                                      

                     3 —                                               1. 6Н     2. 5Н      3. 36Н      4. 30Н      

                     2 —                                   

                     1 —                                    

                     0                                      

                            1     2    3   4     5    6           t,с      

7. Автомобиль движется со скоростью V1 = 90 км/ч, а мотоцикл со скоростью

V2 = 180 км/ч. Масса мотоцикла m = 500 кг. Отношение импульса автомобиля к импульсу мотоцикла равно 2. Масса автомобиля равна

1) 2000 кг             2) 3000 кг                    3) 4000 кг            4) 8000 кг                  

8. Атом массой m, движущийся со скоростью V, столкнулся с неподвижным атомом массой 2m. Каким суммарным импульсом обладают два атома после столкновения?

1) mV/З                2) mV/2                       3) mV                           4) 3mV            

9. Два космических тела сближаются. Как изменится сила гравитационного притяжения одного тела к другому при уменьшении расстояния между телами в 1,5 раза по сравнению с первоначальным?

1) уменьшается в 1,5 раза        3) увеличивается в 1,5 раза

2) уменьшается в 2,25 раза      4) увеличивается в 2,25 раза                  

10. Мяч массой m бросают горизонтально с балкона. Если пренебречь сопротивлением воздуха, то при спуске мяча на высоту h относительно первоначального положения, его полная механическая энергия

1) увеличится на величину mgh      2) уменьшится на величину mgh                

3) не изменится                 4) будет неизвестна, так как не задана начальная скорость

11. Как изменится потенциальная энергия тела, если при неизменной высоте над землей массу тела уменьшить в 2 раза

1) увеличится в 4 раза                2) увеличится в 2 раза

3) уменьшится  в 2 раза              4) не изменится                                          

12. Как изменится кинетическая энергия тела, если при неизменной массе тела его скорость уменьшить в 3 раза?

1) увеличится в 9 раз                2) уменьшится в 9 раз

3) уменьшится  в 3 раза              4) не изменится                                          

В1. Тело массой 10кг падает в воздухе с ускорением 2 м/с2, величина силы сопротивления воздуха постоянна. Вычислить значение силы сопротивления воздуха. Чертеж обязателен.

В2. Пластилиновый шарик массой 50 г налетает на покоящийся пластилиновый шарик массой 150 г. После удара шарики движутся вместе со скоростью 1 м/с. Определить скорость движения налетающего шарика. Чертеж обязателен.

С1. С каким ускорением тело весом 800 Н погружается в воду, если выталкивающая сила,

действующая на него равна 300 Н, а сила сопротивления воды 180 Н? Чертеж обязателен.

С2. Мост, прогибаясь под тяжестью поезда массой 400 т, образует дугу радиусом 2000 м. Определить силу давления поезда в середине моста, если скорость поезда равна 20 м/с.

Контрольная работа по теме «Динамика» 9 класс

Вариант 5

1. Нить, привязанная одним концом к вбитому в стену гвоздю, разорвется, если другой ее конец тянуть с силой не менее 50Н. Чему равно наименьшее значение сил, с которыми растягивают эту же нить за оба конца, при котором она разорвется?

1) 25Н        2) 50Н      3) 75Н       4) 100Н

2. Как будет двигаться тело массой 4 кг под действием постоянной силы 16 Н?

1. равномерно со скоростью 4 м/с

2. равномерно со скоростью 0,25 м/с

3. равноускоренно с ускорением 4 м/с2

4. равноускоренно с ускорением 0,25 м/с2                                    

3. В инерциальной системе отсчета сила F сообщает телу массой m ускорение a. Как изменится ускорение тела, если массу тела в 2 раза увеличить, а действующую на него силу вдвое уменьшить?

1) не изменится  2) увеличится в 4 раза  

3) уменьшится в 8 раз  4) уменьшится в 4 раза                  

4. Тело, брошенное под углом к горизонту, движется по криволинейной траектории. Если сопротивление воздуха пренебрежимо мало, и в точке А этой траектории вектор скорости тела имеет направление по стрелке 1 на рисунке, то вектор его ускорения имеет направление, указанное стрелкой

1) 1          2) 2          3)3          4) 4        

5. На рисунке представлены три вектора сил, приложенных к

одной точке и лежащих в одной плоскости. Модуль вектора силы

F1 равен 4 Н. Модуль равнодействующей векторов F1, F2  и F3 равен

1) 9 Н      2) 7 Н         3) 5 Н           4) 1 Н            

6.               ,м/с                              Скорость движущегося вдоль оси ОХ тела массой 50 кг

                     7 —                                  изменяется со временем в соответствии с графиком

                     6 —                                 (см. рисунок). Определите силу, действующую на тело

                     5 —                                        

                     4 —                                      

                     3 —                                               1. 25Н     2. 100Н      3. 200Н      4. 125Н      

                     2 —                                   

                     1 —                                    

                     0                                      

                            1     2    3   4     5    6           t,с      

7. Танк движется со скоростью V1= 18 км/ч, а грузовик со скоростью V2= 72 км/ч, Масса танка m = 36000 кг. Отношение импульса танка к импульсу грузовика равно 2,25. Масса грузовика равна

1) 1500 кг         2) 3000 кг                      3) 4000 кг                      4) 8000 кг                

8. Человек массой 40 кг прыгает из неподвижной тележки массой 60 кг с горизонтальной скоростью 3 м/с относительно лодки. Если сопротивление пренебрежимо мало, то тележка после прыжка человека движется относительно Земли со скоростью

1) 3 м/с                  2) 2 м/с                      3) 1,5 м/с             4) 1 м/с                        

9. Как измениться сила притяжения двух космических тел, если при неизменном расстоянии между ними массу каждого из них увеличить в 2 раза?

1) увеличится в 2 раза        3) уменьшится в 2 раза  

2) увеличится в 4 раза          4) уменьшится в 4 раза              

10. Ведро массой m поднимают на веревке вертикально вверх с постоянной скоростью. Когда ведро поднимется на высоту h относительно первоначального положения, его полная механическая энергия

1) увеличится на величину mgh             2) уменьшится на величину mgh

3) не изменится                             4) будет неизвестна, так как не задана скорость      

11. Как изменится потенциальная энергия тела, если высоту над землей и массу тела увеличить в 2 раза

1) увеличится в 4 раза                2) увеличится в 2 раза

3) уменьшится  в 2 раза              4) не изменится                                          

12. Как изменится кинетическая энергия тела, если при неизменной скорости тела его массу уменьшить в 2 раза?

1) увеличится в 2 раза                2) уменьшится в 4 раза

3) уменьшится  в 2 раза              4) не изменится                                          

В1. Тело массой 2 кг поднимают на веревке вертикально вверх с ускорением 0,5 . Чему равна сила натяжения веревки? Чертеж обязателен.

В2. Стоящий на льду человек массой 50 кг бросает предмет массой 0,5 кг в горизонтальном направлении со скоростью 10 м/с. С какой скоростью человек начнет скользить по льду? Чертеж обязателен.

С1. Тело массой 0,5кг всплывает в воде с ускорением. С каким ускорением всплывает тело, если сила сопротивления движению равна 14Н, а сила Архимеда 20Н? Чертеж обязателен.

С2. Самолет делает «мертвую петлю» радиусом 100 м и движется по ней со скоростью 280 км/ч. С какой силой летчик массой 890 кг будет давить на сидение самолета в самой верхней точке петли? Чертеж обязателен.

Контрольная работа по теме «Динамика» 9 класс

Вариант 6

1. Вода массой М находится в сосуде массой m. Сравните силу действия воды на дно сосуда с силой действия дна сосуда на воду F2.

2. Как будет двигаться тело массой 6 кг под действием постоянной силы 24 Н?

1) равномерно со скоростью 4 м/с            2) равномерно со скоростью 0,25 м/с

3) равноускоренно с ускорением 4 м/с2  4) равноускоренно с ускорением 0,25 м/с2

3. В инерциальной системе отсчета сила F, действуя на тело массы m, сообщает ему ускорение а. Как надо изменить массу тела, чтобы, увеличив силу вдвое, увеличить его ускорение в 4 раза?

1) увеличить в 2 раза                                        2) уменьшить в 2 раза

3) увеличить в 8 раз                                         4) уменьшить в 8 раз                          

4. Тело, брошенное под углом к горизонту, снижется по криволинейной траектории. Если сопротивление воздуха пренебрежимо мало, и в точке А этой траектории вектор скорости тела имеет направление по стрелке 4 на рисунке, то вектор его ускорения имеет направление, указанное стрелкой

1) 1                   2) 2                         3) 3                          4) 4     

5. На рисунке представлены три вектора сил, приложенных к одной точке и лежащих в одной плоскости. Модуль вектора силы F1 равен 3 Н. Модуль равнодействующей векторов F1, F2 и F3 равен

1) 9Н       2) 8 Н       3) 6 Н          4) 0 Н        4

6.               ,м/с                             Скорость движущегося вдоль оси ОХ тела массой 40 кг

                     7 —                                 изменяется со временем в соответствии с графиком

                     6 —                                 (см. рисунок). Определите силу, действующую на тело

                     5 —                                        

                     4 —                                    

                     3 —                                              1. 40Н     2. 160Н      3. 10Н      4. 30Н    

                     2 —                                    

                     1 —                                    

                     0                                      

                            1     2    3   4     5    6           t,с      

7. Самолет летит со скоростью V1 = 180 км/ч, а вертолет со скоростью V2= 90 км/ч. Масса самолета m= 3000 кг. Отношение импульса caмолета к импульсу вертолета равно 1,5. Масса вертолета равна

1) 1500 кг          2) 3000 кг                      3) 4000 кг                      4) 8000 кг            

8. Человек массой 50 кг прыгает из неподвижной лодки массой 50 кг на берег с горизонтальной скоростью 4 м/с относительно лодки. Если сопротивление воды движению лодки пренебрежимо мало, то лодка после прыжка человека движется относительно Земли со скоростью

1) 4 м/с                  2) 3 м/с                      3) 2 м/с             4) 1 м/с                      

9. Как измениться сила притяжения между двумя шариками, если при неизменном расстоянии между ними массу каждого из них увеличить в 3 раза?

1) увеличивается в 3 раза     3) уменьшается в 3 раза

2) увеличивается в 9 раз       4) уменьшается в 9 раз                                      

10. Ведро массой m опускают на веревке вертикально вниз с постоянной скоростью. Когда ведро опустится на высоту h относительно первоначального положения, его полная механическая энергия

1) увеличится на величину mgh             2) уменьшится на величину mgh

3) не изменится                                   4) будет неизвестна, так как не задана скорость  

11. Как изменится потенциальная энергия тела, если высоту над землей и массу тела увеличить в 3 раза

1) увеличится в 3 раза                2) увеличится в 9 раз

3) уменьшится  в 9 раз              4) не изменится                                          

12. Как изменится кинетическая энергия тела, если при неизменной массе тела его скорость уменьшить в 2 раза?

1) увеличится в 4 раза                2) уменьшится в 4 раза

3) уменьшится  в 2 раза              4) не изменится                                          

В1. Тело массой 20 кг поднимают на веревке вертикально вверх, приложив силу 220 Н. С каким ускорением движется тело? Чертеж обязателен.

В2. Снаряд массой 20 кг, летящий горизонтально со скоростью 500 м/с, попадает в неподвижную платформу с песком массой 10 т и застревает в песке. С какой скоростью стала двигаться платформа? Чертеж обязателен.

С1. На тело, лежащее на горизонтальной плоскости, действует сила 20 Н. Тело движется с ускорением 8,5 м/. Масса тела 2 кг. Найдите величину силы трения. Чертеж обязателен.

С2. Мальчик массой 50 кг качается на качелях с длиной подвеса 4 м. С какой силой он давит на сидение при прохождении среднего положения со скоростью 6 м/с? Чертеж обязателен.

Контрольная работа по теме «Динамика» 9 класс

Вариант 7

1. Через неподвижный блок перекинута невесомая нерастяжимая нить, к концам которой подвешены грузы равной массы m. Чему равна сила натяжения нити?

1) 0,25mg         2) 0,5mg       3) mg      4) 2mg

2. Что будет с модулем скорости движущегося тела, если к нему приложить силы, равнодействующая которых равна 0?

1) будет увеличиваться         2) будет уменьшаться                    3) останется прежней

4) в начале увеличится, а затем уменьшится                                            

3. В инерциальной системе отсчета сила F, действуя на тело массы m, сообщает ему ускорение а. Как надо изменить силу, чтобы, увеличив массу вдвое, увеличить его ускорение в 4 раза?

1) увеличить в 2 раза                                        2) уменьшить в 2 раза

3) увеличить в 8 раз                                         4) уменьшить в 8 раз                            

4. Самолет выполняет фигуру высшего пилотажа «мертвая петля». Как направлен вектор ускорения самолета в тот момент времени когда вектор равнодействующей всех сил направлен вертикально вниз к центру окружности, а вектор скорости самолета направлен горизонтально?

1) Вертикально вверх                                 2) По направлению вектора скорости

3) Противоположно вектору скорости      4) Вертикально вниз                                

5. На тело действуют четыре силы (см. рис.)  F1=10Н,      F2=6Н,      F3=3Н,    F4=6Н

Чему равна равнодействующая этих сил?

                                                                                      F2                                

                                                        F1                                                       F3

                                                                                  F4

1) 25Н    2) 7Н    3) 19Н    4) 13Н          

6.         ,м/с                                Скорость движущегося вдоль оси ОХ тела массой 4 кг

                     7 —                              изменяется со временем в соответствии с графиком

                     6 —                            (см. рисунок). Определите силу, действующую на тело

                     5 —                                        

                      4 —                                      

                     3 —                                               1. 24Н     2. 8Н      3. 2Н      4. 4Н      

                     2 —                    

                     1 —                                    

                     0                                      

                               2   4     6    8   10   12           t,с          

7. Поезд движется со скоростью V1 = 90 км/ч, а теплоход со скоростью V2= 36 км/ч. Масса поезда m = 100 тонн. Отношение импульса поезда к импульсу теплохода равно 5. Масса теплохода равна

1) 20 тонн           2) 50 тонн                      3) 100 тонн                    4) 200 тонн              

8. Вагон массой 2m, движущийся со скоростью V, сталкивается с неподвижным вагоном массой 4m. Каким суммарным импульсом обладают два вагона после столкновения? Взаимодействие вагонов с другими телами пренебрежимо мало.

1) 0                     2) 6mV                       3) 2mV                      4) mV/6                  

9. Два космических тела сближаются. Как изменится сила гравитационного притяжения одного тела к другому при увеличении расстояния между телами в 1,5 раза по сравнению с первоначальным?

1) уменьшается в 1,5 раза        3) увеличивается в 1,5 раза

2) уменьшается в 2,25 раза      4) увеличивается в 2,25 раза                                  

10. Шар массой m движется по окружности радиуса R в горизонтальной плоскости со скоростью V. Когда шар сделает один полный оборот, его полная механическая энергия

1) увеличится на величину mgR         2) увеличится на величину /2

3) уменьшится на величину mgR            4) не изменится                                

11. Строитель поднял кирпич массой 5 кг на высоту 2м. Что произошло с потенциальной энергией кирпича?

1) увеличилась на 10Дж                         2) увеличилась на 100Дж                            

3) уменьшилась на 100Дж            4) будет неизвестна, так как не задана скорость  

12. Какой кинетической энергией обладает тело массой 10кг, движущееся со скоростью 36км/ч?

1) 500Дж     2) 6480Дж     3) 180Дж    4) 360Дж                      

В1. Тело поднимают на веревке вертикально вверх с ускорением 2, приложив силу

120 Н.. Чему равна масса тела? Чертеж обязателен.

В2. Вагон массой 100 т, движущийся со скоростью 2 м/с по горизонтальной дороге, сталкивается с неподвижной платформой массой 20 т. Чему равна скорость движения сцепки? Чертеж обязателен.

C1. Тело массой 5 кг с помощью каната начинают равноускоренно поднимают вертикально вверх. Чему равна сила, действующая на тело со стороны каната, если известно, что за 3с груз был поднят на высоту 12м? Чертеж обязателен.

С2. Мотоцикл массой 500 кг проезжает овраг со скоростью 20 м/с в нижней его точке. Чему равен радиус оврага, если вес мотоцикла в этот момент равен 7 кН? Чертеж обязателен.

Ответы контрольной работы по теме «Динамика» 9 класс

1

2

3

4

5

6

7

8

9

10

11

12

В1

3

4

3

2

2

2

1

3

2

4

2

1

В2

3

3

3

2

3

2

4

2

4

2

3

2

В3

3

2

2

1

2

4

4

4

4

3

2

3

В4

3

4

1

2

2

2

1

3

4

3

3

2

В5

2

3

4

2

3

1

3

2

2

1

1

3

В6

3

3

2

2

4

4

3

1

2

2

2

2

В7

3

3

3

4

2

3

2

2

2

4

2

1

Часть В    4 балла            Часть С  5 баллов

Оценка  3         7-10        

               4         11-14

               5         более 15

3.4 Движение с постоянным ускорением — Университетская физика, том 1

Цели обучения

К концу этого раздела вы сможете:

  • Определите, какие уравнения движения следует использовать для решения неизвестных.
  • Используйте соответствующие уравнения движения для решения задачи преследования двух тел.

Можно предположить, что чем больше ускорение, скажем, у автомобиля, удаляющегося от знака «Стоп», тем больше смещение автомобиля за данный момент времени.Но мы не разработали конкретное уравнение, которое связывает ускорение и смещение. В этом разделе мы рассмотрим некоторые удобные уравнения кинематических отношений, начиная с определений смещения, скорости и ускорения. Сначала мы исследуем движение одного объекта, называемого движением одного тела. Затем мы исследуем движение двух объектов, называемых задачами преследования двух тел .

Обозначение

Сначала сделаем несколько упрощений в обозначениях.Принятие начального времени равным нулю, как если бы время измерялось секундомером, является большим упрощением. Поскольку прошедшее время

, принимая

означает, что

, последнее время на секундомере. Когда начальное время принимается равным нулю, мы используем индекс 0 для обозначения начальных значений положения и скорости. То есть

— начальная позиция и

— начальная скорость .Мы не ставим индексы на окончательные значения. То есть t — это конечный момент времени , x — конечная позиция , а v — конечная скорость . Это дает более простое выражение для прошедшего времени:

.

. Это также упрощает выражение для смещения x , которое теперь составляет

. Кроме того, это упрощает выражение для изменения скорости, которое теперь составляет

.

. Подводя итог, используя упрощенные обозначения, с начальным временем, принятым равным нулю,

, где нижний индекс 0 обозначает начальное значение, а отсутствие нижнего индекса означает конечное значение в любом рассматриваемом движении.

Теперь мы делаем важное предположение, что ускорение постоянно . Это предположение позволяет нам избегать использования расчетов для определения мгновенного ускорения. Поскольку ускорение постоянно, среднее и мгновенное ускорения равны, то есть

Таким образом, мы можем использовать символ a для ускорения в любое время. Предположение, что ускорение является постоянным, не серьезно ограничивает ситуации, которые мы можем изучить, и не ухудшает точность нашего лечения.Во-первых, ускорение равно постоянно в большом количестве ситуаций. Кроме того, во многих других ситуациях мы можем точно описать движение, приняв постоянное ускорение, равное среднему ускорению для этого движения. Наконец, для движения, во время которого ускорение резко меняется, например, когда автомобиль разгоняется до максимальной скорости, а затем тормозит до остановки, движение можно рассматривать в отдельных частях, каждая из которых имеет собственное постоянное ускорение.

Смещение и положение от скорости

Чтобы получить наши первые два уравнения, мы начнем с определения средней скорости:

Замена

упрощенным обозначением

и

дает

Решение для x дает нам

, где средняя скорость

Уравнение

отражает тот факт, что при постоянном ускорении v — это просто среднее значение начальной и конечной скоростей.(Рисунок) графически иллюстрирует эту концепцию. В части (а) рисунка ускорение является постоянным, а скорость увеличивается с постоянной скоростью. Средняя скорость на 1-часовом интервале от 40 км / ч до 80 км / ч составляет 60 км / ч:

В части (b) ускорение не является постоянным. В течение 1-часового интервала скорость ближе к 80 км / ч, чем к 40 км / ч. Таким образом, средняя скорость больше, чем в части (а).

Рисунок 3.18 (a) График зависимости скорости от времени с постоянным ускорением, показывающий начальную и конечную скорости

.Средняя скорость

. (б) График зависимости скорости от времени с изменением ускорения со временем. Средняя скорость не указана в

, но больше 60 км / ч.

Решение окончательной скорости по ускорению и времени

Мы можем вывести еще одно полезное уравнение, манипулируя определением ускорения:

Замена

упрощенным обозначением

и

дает нам

Решение для v дает

Пример

Расчет конечной скорости

Самолет приземляется с начальной скоростью 70.0 м / с, а затем замедляется со скоростью 1,50 м / с 2 на 40,0 с. Какова его конечная скорость?

Стратегия

Сначала мы идентифицируем известные:

.

Во-вторых, мы идентифицируем неизвестное; в данном случае это конечная скорость

.

Наконец, мы определяем, какое уравнение использовать. Для этого мы выясняем, какое кинематическое уравнение дает неизвестное в терминах известных. Мы рассчитываем окончательную скорость, используя (Рисунок),

.

Решение

[Показать-ответ q = ”287818 ″] Показать ответ [/ Показать-ответ]
[hidden-answer a =” 287818 ″] Подставить известные значения и решить:

(рисунок) — это эскиз, на котором показаны векторы ускорения и скорости. [/ Hidden-answer]

Рис. 3.19. Самолет приземляется с начальной скоростью 70,0 м / с и замедляется до конечной скорости 10,0 м / с, прежде чем направиться к терминалу. Обратите внимание, что ускорение отрицательное, потому что его направление противоположно его скорости, которая положительна.
Значение

Конечная скорость намного меньше начальной скорости, требуемой при замедлении, но все же положительная (см. Рисунок). С реактивными двигателями обратная тяга может поддерживаться достаточно долго, чтобы остановить самолет и начать движение назад, на что указывает отрицательная конечная скорость, но в данном случае это не так.

Уравнение

не только помогает при решении задач.

дает нам представление о взаимосвязи между скоростью, ускорением и временем.Мы видим, например, что

  • Конечная скорость зависит от того, насколько велико ускорение и как долго оно длится
  • Если ускорение равно нулю, то конечная скорость равна начальной скорости ( v = v 0 ), как и ожидалось (другими словами, скорость постоянна)
  • Если a отрицательное, то конечная скорость меньше начальной скорости

Все эти наблюдения соответствуют нашей интуиции. Обратите внимание, что всегда полезно исследовать основные уравнения в свете нашей интуиции и опыта, чтобы убедиться, что они действительно точно описывают природу.

Решение для конечного положения с постоянным ускорением

Мы можем объединить предыдущие уравнения, чтобы найти третье уравнение, которое позволяет нам вычислить окончательное положение объекта, испытывающего постоянное ускорение. Начнем с

Добавление

в каждую сторону этого уравнения и деление на 2 дает

с

для постоянного разгона, имеем

Теперь подставим это выражение вместо

в уравнение для смещения,

, давая

Пример

Расчет смещения ускоряющегося объекта

Драгстеры могут развивать среднее ускорение 26.0 м / с 2 . Предположим, драгстер ускоряется из состояния покоя в течение 5,56 с (рисунок). Как далеко он пролетит за это время?

Рисунок 3.20. Пилот Top Fuel американской армии Тони «Сержант» Шумахер начинает гонку с контролируемого выгорания. (Источник: подполковник Уильям Термонд. Фотография предоставлена ​​армией США.)
Стратегия

Сначала нарисуем эскиз (рисунок). Нас просят найти смещение, которое составляет x , если мы возьмем

равняется нулю.(Подумайте о

как стартовая линия гонки. Он может быть где угодно, но мы называем его нулем и измеряем все остальные положения относительно него.) Мы можем использовать уравнение

, когда мы идентифицируем

,

, и т. из постановки задачи.

Рис. 3.21 Эскиз разгоняющегося драгстера.
Решение

[show-answer q = ”9 ″] Показать ответ [/ show-answer]
[hidden-answer a =” 9 ″] Во-первых, нам нужно определить известные.Запуск из состояния покоя означает, что

, a равно 26,0 м / с2, а t равно 5,56 с.
Во-вторых, мы подставляем известные значения в уравнение, чтобы найти неизвестное:

Поскольку начальное положение и скорость равны нулю, это уравнение упрощается до

Подстановка идентифицированных значений a и t дает

[/ hidden-answer]

Значение

Если мы переведем 402 м в мили, мы обнаружим, что пройденное расстояние очень близко к четверти мили, стандартному расстоянию для дрэг-рейсинга.Итак, наш ответ разумный. Это впечатляющий водоизмещение всего за 5,56 с, но первоклассные драгстеры могут преодолеть четверть мили даже за меньшее время. Если бы драгстеру была присвоена начальная скорость, это добавило бы еще один член в уравнение расстояния. Если в уравнении использовать те же ускорение и время, пройденное расстояние будет намного больше.

Что еще мы можем узнать, исследуя уравнение

Мы видим следующие отношения:

  • Смещение зависит от квадрата прошедшего времени, когда ускорение не равно нулю.На (Рис.) Драгстер преодолевает только четверть общего расстояния за первую половину прошедшего времени.
  • Если ускорение равно нулю, то начальная скорость равна средней скорости.

    и

Решение окончательной скорости с расстояния и ускорения

Четвертое полезное уравнение может быть получено путем другой алгебраической обработки предыдущих уравнений. Если мы решим

за т , получаем

Подставляя это и

в

, получаем

Пример

Расчет конечной скорости

Рассчитайте окончательную скорость драгстера (рисунок) без использования информации о времени.

Стратегия

Уравнение

идеально подходит для этой задачи, поскольку он связывает скорости, ускорение и смещение и не требует информации о времени.

Решение

[show-answer q = ”350935 ″] Показать ответ [/ show-answer]
[hidden-answer a =” 350935 ″] Сначала мы идентифицируем известные значения. Мы знаем, что v0 = 0, поскольку драгстер стартует из состояния покоя. Мы также знаем, что x — x0 = 402 м (это был ответ на (Рисунок)).Среднее ускорение составило a = 26,0 м / с2.

ПЕРЕРЫВОВ Во-вторых, мы подставляем известные в уравнение

и решите относительно v:

ПЕРЕРЫВ

Таким образом, ПЕРЕРЫВ

[/ hidden-answer]

Значение

Скорость 145 м / с составляет около 522 км / ч или около 324 миль / ч, но даже эта головокружительная скорость не достигает рекорда для четверти мили. Также обратите внимание, что квадратный корень имеет два значения; мы взяли положительное значение, чтобы указать скорость в том же направлении, что и ускорение.

Исследование уравнения

может дать дополнительную информацию об общих отношениях между физическими величинами:

  • Конечная скорость зависит от величины ускорения и расстояния, на котором оно действует.
  • При фиксированном ускорении автомобиль, который едет вдвое быстрее, просто не останавливается на удвоенном расстоянии. Чтобы остановиться, нужно гораздо дальше. (Вот почему у нас есть зоны с пониженной скоростью возле школ.)

Объединение уравнений

В следующих примерах мы продолжаем исследовать одномерное движение, но в ситуациях, требующих немного большего количества алгебраических манипуляций.Примеры также дают представление о методах решения проблем. Следующее примечание предназначено для облегчения поиска необходимых уравнений. Имейте в виду, что эти уравнения не являются независимыми. Во многих ситуациях у нас есть два неизвестных, и нам нужно два уравнения из набора для решения для неизвестных. Для решения данной ситуации нам нужно столько уравнений, сколько неизвестных.

Сводка кинематических уравнений (постоянная a )

Прежде чем мы перейдем к примерам, давайте более внимательно рассмотрим некоторые уравнения, чтобы увидеть поведение ускорения при экстремальных значениях.Переставляя (рисунок), получаем

Отсюда мы видим, что в течение конечного времени, если разница между начальной и конечной скоростями мала, ускорение невелико, приближаясь к нулю в том пределе, когда начальная и конечная скорости равны. Напротив, в лимите

для конечной разницы между начальной и конечной скоростями ускорение становится бесконечным.

Аналогичным образом, переставляя (рисунок), мы можем выразить ускорение в терминах скоростей и смещения:

Таким образом, при конечной разнице между начальной и конечной скоростями ускорение становится бесконечным, в пределе смещение приближается к нулю.Ускорение приближается к нулю в пределе, разница в начальной и конечной скоростях приближается к нулю для конечного смещения.

Пример

Как далеко уезжает машина?

На сухом бетоне автомобиль может замедляться со скоростью 7,00 м / с 2 , тогда как на мокром бетоне он может замедляться только со скоростью 5,00 м / с 2 . Найдите расстояния, необходимые для остановки автомобиля, движущегося со скоростью 30,0 м / с (около 110 км / ч) по (а) сухому бетону и (б) мокрому бетону. (c) Повторите оба вычисления и найдите смещение от точки, где водитель видит, что светофор становится красным, принимая во внимание время его реакции, равное 0.500 с, чтобы нажать на педаль тормоза.

Стратегия

Для начала нам нужно нарисовать эскиз (рисунок). Чтобы определить, какие уравнения лучше всего использовать, нам нужно перечислить все известные значения и точно определить, что нам нужно решить.

Рис. 3.22 Образец эскиза для визуализации замедления и тормозного пути автомобиля.
Решение
  1. Во-первых, нам нужно определить известные и то, что мы хотим решить. Мы знаем, что v 0 = 30.0 м / с, v = 0 и a = −7,00 м / с 2 ( a отрицательно, потому что оно находится в направлении, противоположном скорости). Возьмем x 0 равным нулю. Ищем перемещение

    или x x 0 . Во-вторых, мы определяем уравнение, которое поможет нам решить проблему. Лучшее уравнение для использования —

    Это уравнение лучше всего, потому что оно включает только одно неизвестное, x .Нам известны значения всех других переменных в этом уравнении. (Другие уравнения позволили бы нам решить для x , но они требуют, чтобы мы знали время остановки, t , которое мы не знаем. Мы могли бы их использовать, но это потребовало бы дополнительных вычислений.)

    В-третьих, мы изменим уравнение, чтобы найти x :

    и подставьте известные значения:

    Таким образом,

  2. Эта часть может быть решена точно так же, как (а).Единственное отличие состоит в том, что ускорение составляет −5,00 м / с 2 . Результат

  3. [show-answer q = ”175639 ″] Показать ответ [/ show-answer]
    [hidden-answer a =” 175639 ″] Когда водитель реагирует, тормозной путь такой же, как в пунктах (a) и ( б) для сухого и влажного бетона. Итак, чтобы ответить на этот вопрос, нам нужно вычислить, как далеко проехал автомобиль за время реакции, а затем добавить это время ко времени остановки. Разумно предположить, что скорость остается постоянной в течение времени реакции водителя.Для этого мы, опять же, определяем известные и то, что мы хотим решить. Мы знаем это

    ,

    и

    . Берем

    равняется нулю. Ищем

    . Во-вторых, как и раньше, мы определяем лучшее уравнение для использования. В данном случае

    работает хорошо, потому что единственное неизвестное значение — это x, которое мы и хотим найти.В-третьих, мы подставляем известные, чтобы решить уравнение:

    Это означает, что автомобиль движется на 15,0 м, пока водитель реагирует, в результате чего общее смещение в двух случаях с сухим и мокрым бетоном на 15,0 м больше, чем при мгновенной реакции. Наконец, мы добавляем смещение во время реакции к смещению при торможении ((Рисунок)),

    и находят (а) равным 64,3 м + 15,0 м = 79,3 м в сухом состоянии и (б) равным 90,0 м + 15,0 м = 105 м во влажном состоянии.[/ hidden-answer]

Рисунок 3.23 Расстояние, необходимое для остановки автомобиля, сильно варьируется в зависимости от дорожных условий и времени реакции водителя. Здесь показаны значения тормозного пути для сухого и мокрого покрытия, рассчитанные в этом примере для автомобиля, движущегося со скоростью 30,0 м / с. Также показано общее расстояние, пройденное от точки, когда водитель впервые видит, что свет загорается красным, при условии, что время реакции составляет 0,500 с.
Значение

Смещения, найденные в этом примере, кажутся разумными для остановки быстро движущегося автомобиля.Остановка автомобиля на мокром асфальте должна длиться дольше, чем на сухом. Интересно, что время реакции значительно увеличивает смещения, но более важен общий подход к решению проблем. Мы идентифицируем известные и определяемые величины, а затем находим соответствующее уравнение. Если существует более одного неизвестного, нам нужно столько независимых уравнений, сколько неизвестных необходимо решить. Часто есть несколько способов решить проблему. Фактически, различные части этого примера могут быть решены другими методами, но представленные здесь решения являются самыми короткими.

Пример

Расчет времени

Предположим, автомобиль выезжает на шоссе на съезде длиной 200 м. Если его начальная скорость равна 10,0 м / с, а он ускоряется со скоростью 2,00 м / с 2 , сколько времени потребуется автомобилю, чтобы преодолеть 200 м по рампе? (Такая информация может быть полезна транспортному инженеру.)

Стратегия

Сначала рисуем эскиз (рисунок). Нам предлагается решить за время т . Как и раньше, мы идентифицируем известные величины, чтобы выбрать удобное физическое соотношение (то есть уравнение с одной неизвестной, t .)

Рис. 3.24 Эскиз автомобиля, ускоряющегося на съезде с автострады.
Решение

[show-answer q = ”712029 ″] Показать ответ [/ show-answer]
[hidden-answer a =” 712029 ″] Опять же, мы определяем известные нам и то, что мы хотим решить. Мы знаем, что

, и x = 200 м.

Нам нужно решить для t. Уравнение

работает лучше всего, потому что единственная неизвестная в уравнении — это переменная t, для которой нам нужно решить.Из этого понимания мы видим, что когда мы вводим известные значения в уравнение, мы получаем квадратное уравнение.

Нам нужно изменить уравнение, чтобы найти t, а затем подставить известные значения в уравнение:

Затем мы упрощаем уравнение. Единицы измерения отменяются, потому что они есть в каждом члене. Мы можем получить единицы секунд для отмены, взяв t = t s, где t — величина времени, а s — единица измерения. Остается

Затем мы используем формулу корней квадратного уравнения, чтобы найти t,

, что дает два решения: t = 10.0 и t = -20,0. Отрицательное значение времени неразумно, так как это будет означать, что событие произошло за 20 секунд до начала движения. Мы можем отказаться от этого решения. Таким образом,

[/ hidden-answer]

Значение

Всякий раз, когда уравнение содержит неизвестный квадрат, есть два решения. В некоторых проблемах имеют смысл оба решения; в других случаях разумно только одно решение. Ответ 10,0 с кажется разумным для типичной автострады на съезде.

Проверьте свое понимание

Пилотируемая ракета ускоряется со скоростью 20 м / с 2 во время пуска.Сколько времени нужно, чтобы ракета достигла скорости 400 м / с?

[show-answer q = ”fs-id1168329484424 ″] Показать решение [/ show-answer]

[скрытый-ответ a = ”fs-id1168329484424 ″]

Чтобы ответить на этот вопрос, выберите уравнение, которое позволяет нам решить для времени t , учитывая только a , v 0 и v :

Перегруппировать для решения для т :

[/ hidden-answer]

Пример

Ускорение космического корабля

Космический корабль покинул орбиту Земли и направляется к Луне.Разгоняется со скоростью 20 м / с 2 за 2 мин и преодолевает расстояние в 1000 км. Каковы начальная и конечная скорости космического корабля?

Стратегия

Нас просят найти начальную и конечную скорости космического корабля. Глядя на кинематические уравнения, мы видим, что одно уравнение не дает ответа. Мы должны использовать одно кинематическое уравнение для решения одной из скоростей и подставить его в другое кинематическое уравнение, чтобы получить вторую скорость. Таким образом, мы решаем два кинематических уравнения одновременно.

Решение

[show-answer q = ”835228 ″] Показать ответ [/ show-answer]
[hidden-answer a =” 835228 ″] Сначала мы решаем для

с использованием

Затем подставляем

в

, чтобы найти окончательную скорость:

[/ hidden-answer]

Значение

Есть шесть переменных: смещение, время, скорость и ускорение, которые описывают движение в одном измерении.Начальные условия данной задачи могут быть множеством комбинаций этих переменных. Из-за такого разнообразия решения могут быть нелегкими, например простой заменой в одно из уравнений. Этот пример показывает, что решения кинематики могут потребовать решения двух одновременных кинематических уравнений.

Освоив основы кинематики, мы можем перейти ко многим другим интересным примерам и приложениям. В процессе разработки кинематики мы также увидели общий подход к решению проблем, который дает как правильные ответы, так и понимание физических взаимоотношений.Следующий уровень сложности в наших задачах кинематики связан с движением двух взаимосвязанных тел, называемых задачами преследования двух тел .

Задачи преследования двух тел

До этого момента мы рассматривали примеры движения с участием одного тела. Даже для задачи с двумя автомобилями и тормозным путем на мокрой и сухой дороге мы разделили эту задачу на две отдельные задачи, чтобы найти ответы. В задаче преследования двух тел движения объектов связаны — это означает, что неизвестное, которое мы ищем, зависит от движения обоих объектов.Чтобы решить эти проблемы, мы пишем уравнения движения для каждого объекта, а затем решаем их одновременно, чтобы найти неизвестное. Это проиллюстрировано на (Рисунок).

Рис. 3.25 Сценарий преследования с двумя телами, в котором автомобиль 2 имеет постоянную скорость, а автомобиль 1 идет сзади с постоянным ускорением. Автомобиль 1 догонит автомобиль 2 позже.

Время и расстояние, необходимое для того, чтобы автомобиль 1 догнал автомобиль 2, зависят от начального расстояния, на которое автомобиль 1 находится от автомобиля 2, а также от скорости обоих автомобилей и ускорения автомобиля 1.Чтобы найти эти неизвестные, необходимо решить кинематические уравнения, описывающие движение обеих машин.

Рассмотрим следующий пример.

Пример

Гепард ловит газель

Гепард прячется за кустом. Гепард замечает пробегающую мимо газель со скоростью 10 м / с. В тот момент, когда газель проходит мимо гепарда, гепард из состояния покоя ускоряется со скоростью 4 м / с 2 , чтобы поймать газель. а) Сколько времени требуется гепарду, чтобы поймать газель? б) Что такое смещение газели и гепарда?

Стратегия

Мы используем систему уравнений для постоянного ускорения, чтобы решить эту проблему.Поскольку есть два движущихся объекта, у нас есть отдельные уравнения движения, описывающие каждое животное. Но то, что связывает уравнения, — это общий параметр, который имеет одинаковое значение для каждого животного. Если мы внимательно посмотрим на проблему, становится ясно, что общим параметром для каждого животного является их положение x , позднее t . Поскольку они оба начинаются с

, их водоизмещения такие же, в более позднее время т. , когда гепард догоняет газель.Если мы выберем уравнение движения, которое решает смещение для каждого животного, мы можем затем установить уравнения, равные друг другу, и решить для неизвестного, то есть времени.

Решение
  1. [показать-ответ q = ”699945 ″] Показать ответ [/ раскрыть-ответ]
    [скрытый-ответ a =” 699945 ″] Уравнение для газели: газель имеет постоянную скорость, которая является ее средней скоростью, поскольку это не ускоряется. Поэтому мы используем (рисунок) с

    :

    Уравнение для гепарда: гепард ускоряется из состояния покоя, поэтому мы используем (рисунок) с

    .

    и

    :

    Теперь у нас есть уравнение движения для каждого животного с общим параметром, который можно исключить, чтобы найти решение.В этом случае мы решаем для t:

    Газель имеет постоянную скорость 10 м / с, что является ее средней скоростью. Ускорение гепарда составляет 4 м / с2. Оценивая t, время, за которое гепард достигает газели, получаем

    [/ hidden-answer]

  2. [Показать-ответ q = ”316146 ″] Показать ответ [/ Показать-ответ]
    [Скрытый-ответ a =” 316146 ″] Чтобы получить смещение, мы используем уравнение движения гепарда или газели, поскольку они оба должны дать одинаковый ответ.Смещение гепарда:

    Водоизмещение газели:

    Мы видим, что оба смещения равны, как и ожидалось. [/ Hidden-answer]

Значение

Важно анализировать движение каждого объекта и использовать соответствующие кинематические уравнения для описания отдельного движения. Также важно иметь хорошую визуальную перспективу задачи преследования двух тел, чтобы увидеть общий параметр, который связывает движение обоих объектов.

Проверьте свое понимание

Велосипед имеет постоянную скорость 10 м / с. Человек стартует с отдыха и бежит догонять велосипед за 30 с. Какое ускорение у человека?

[show-answer q = ”fs-id1168326827870 ″] Показать решение [/ show-answer]

[скрытый-ответ a = ”fs-id1168326827870 ″]

.
[/ hidden-answer]

Сводка

  • При анализе одномерного движения с постоянным ускорением определите известные величины и выберите соответствующие уравнения для решения неизвестных.Для решения неизвестных требуются одно или два кинематических уравнения, в зависимости от известных и неизвестных величин.
  • Двухчастичные задачи преследования всегда требуют одновременного решения двух уравнений относительно неизвестных.

Концептуальные вопросы

При анализе движения отдельного объекта, какое количество известных физических переменных необходимо для решения неизвестных величин с использованием кинематических уравнений?

Укажите два сценария кинематики одного объекта, в которых три известные величины требуют решения двух кинематических уравнений относительно неизвестных.

[show-answer q = ”fs-id1168326925475 ″] Показать решение [/ show-answer]

[скрытый-ответ a = ”fs-id1168326925475 ″]

Если ускорение, время и смещение являются известными, а начальная и конечная скорости являются неизвестными, то два кинематических уравнения должны решаться одновременно. Также, если конечная скорость, время и смещение являются известными, тогда необходимо решить два кинематических уравнения для начальной скорости и ускорения.

[/ hidden-answer]

Проблемы

Частица движется по прямой с постоянной скоростью 30 м / с.Каково его смещение между t = 0 и t = 5,0 с?

[Показать-ответ q = ”fs-id1168326925504 ″] Показать решение [/ раскрыть-ответ]

[скрытый-ответ a = ”fs-id1168326925504 ″]

150 кв.м

[/ hidden-answer]

Частица движется по прямой с начальной скоростью 30 м / с и постоянным ускорением 30 м / с 2 . Если на

и

, каково положение частицы при t = 5 с?

Частица движется по прямой с начальной скоростью 30 м / с и постоянным ускорением 30 м / с 2 .(а) Какое у него водоизмещение при т = 5 с? б) Какова его скорость в это же время?

[показывать-ответ q = ”fs-id1168326

2 ″] Показать решение [/ показывать-ответ]

[скрытый-ответ a = ”fs-id1168326

2 ″]

а. 525 м;

г.

[/ hidden-answer]

(a) Нарисуйте график зависимости скорости от времени, соответствующий графику перемещения от времени, представленному на следующем рисунке. (b) Определите время или время ( t a , t b , t c и т. д.), при которой мгновенная скорость имеет наибольшее положительное значение. (c) В какое время он равен нулю? (г) В какое время он отрицательный?


[show-answer q = ”966010 ″] Показать ответ [/ show-answer]
[hidden-answer a =” 966010 ″] [/ hidden-answer]

(a) Нарисуйте график зависимости ускорения от времени, соответствующий графику зависимости скорости от времени, представленному на следующем рисунке. (b) Определите время или время ( t a , t b , t c и т. д.), при котором ускорение имеет наибольшее положительное значение. (c) В какое время он равен нулю? (г) В какое время он отрицательный?


[показать-ответ q = ”925936 ″] Показать ответ [/ раскрыть-ответ]

[hidden-answer a = ”925936 ″]

а.

г. Ускорение имеет наибольшее положительное значение на

.

г. Ускорение нулевое на

г. Ускорение отрицательное на

[/ hidden-answer]

Частица имеет постоянное ускорение 6.0 м / с 2 . (а) Если его начальная скорость составляет 2,0 м / с, в какое время его смещение составляет 5,0 м? б) Какова его скорость в то время?

При t = 10 с частица движется слева направо со скоростью 5,0 м / с. При t = 20 с частица движется справа налево со скоростью 8,0 м / с. Предполагая, что ускорение частицы постоянное, определите (а) ее ускорение, (б) ее начальную скорость и (в) момент, когда ее скорость равна нулю.

[показывать-ответ q = ”fs-id1168327148264 ″] Показать решение [/ показывать-ответ]

[скрытый-ответ a = ”fs-id1168327148264 ″]

а.

;
г.

;

г.

[/ hidden-answer]

Хорошо брошенный мяч попадает в рукавицу с хорошей набивкой. Если ускорение мяча

и 1,85 мс

проходит с момента первого прикосновения мяча к рукавице до остановки. Какова начальная скорость мяча?

Пуля в ружье ускоряется от камеры выстрела до конца ствола со средней скоростью

для

.Какова его начальная скорость (то есть конечная скорость)?

[показывать-ответ q = ”fs-id1168329484717 ″] Показать решение [/ показывать-ответ]

[скрытый-ответ a = ”fs-id1168329484717 ″]

[/ hidden-answer]

(a) Пригородный легкорельсовый поезд ускоряется со скоростью 1,35 м / с 2 . Сколько времени нужно, чтобы достичь максимальной скорости 80,0 км / ч, начиная с состояния покоя? (b) Этот же поезд обычно замедляется со скоростью 1,65 м / с 2 .Сколько времени нужно, чтобы остановиться с максимальной скорости? (c) В аварийных ситуациях поезд может замедляться быстрее, останавливаясь на скорости 80,0 км / ч за 8,30 с. Каково его аварийное ускорение в метрах на секунду в квадрате?

При выезде на автостраду автомобиль ускоряется из состояния покоя со скоростью 2,04 м / с 2 за 12,0 с. (а) Нарисуйте набросок ситуации. (б) Перечислите известных в этой проблеме. (c) Как далеко проехал автомобиль за эти 12,0 с? Чтобы решить эту часть, сначала определите неизвестное, а затем укажите, как вы выбрали соответствующее уравнение для его решения.После выбора уравнения покажите свои шаги в поиске неизвестного, проверьте свои единицы и обсудите, является ли ответ разумным. (d) Какова конечная скорость автомобиля? Решите для этого неизвестного таким же образом, как в (c), явно показывая все шаги.

[показывать-ответ q = ”fs-id1168327145386 ″] Показать решение [/ показывать-ответ]

[скрытый-ответ a = ”fs-id1168327145386 ″]

а.

г. Знает:

и

;

г.

, ответ кажется разумным на высоте около 172,8 м; d.

[/ hidden-answer]

Необоснованные результаты В конце забега бегун замедляется со скорости 9,00 м / с со скоростью 2,00 м / с 2 . а) Как далеко она продвинется в следующие 5,00 с? б) Какова ее конечная скорость? (c) Оцените результат. Имеет ли это смысл?

Кровь ускоряется из состояния покоя до 30,0 см / с на расстоянии 1.80 см от левого желудочка сердца. (а) Сделайте набросок ситуации. (б) Перечислите известных в этой проблеме. (c) Сколько времени длится ускорение? Чтобы решить эту часть, сначала определите неизвестное, а затем обсудите, как вы выбрали соответствующее уравнение для его решения. После выбора уравнения покажите свои шаги в решении неизвестного, проверяя свои единицы. (г) Является ли ответ разумным по сравнению со временем биения сердца?

[show-answer q = ”fs-id1168329325655 ″] Показать решение [/ show-answer]

[скрытый-ответ a = ”fs-id1168329325655 ″]

а.

г. Знает:

;

г.

;

г. да

[/ hidden-answer]

Во время удара по воротам хоккеист ускоряет шайбу со скорости 8,00 м / с до 40,0 м / с в том же направлении. Если этот выстрел занимает

, на каком расстоянии разгоняется шайба?

Мощный мотоцикл может разогнаться с места до 26.8 м / с (100 км / ч) всего за 3,90 с. а) Каково его среднее ускорение? б) Как далеко он пролетит за это время?

[show-answer q = ”fs-id1168329293321 ″] Показать решение [/ show-answer]

[скрытый-ответ a = ”fs-id1168329293321 ″]

а. 6,87 с 2 ; б.

[/ hidden-answer]

Грузовые поезда могут развивать только относительно небольшие ускорения. (а) Какова конечная скорость грузового поезда, который ускоряется со скоростью

?

для 8.00 мин, начиная с начальной скорости 4,00 м / с? (б) Если поезд может замедлиться со скоростью

, сколько времени потребуется, чтобы остановиться на этой скорости? (c) Как далеко он продвинется в каждом случае?

Снаряд фейерверка ускоряется из состояния покоя до скорости 65,0 м / с на расстоянии 0,250 м. (а) Рассчитайте ускорение. б) Как долго длилось ускорение?

[показывать-ответ q = ”fs-id1168326954581 ″] Показать решение [/ показывать-ответ]

[скрытый-ответ a = ”fs-id1168326954581 ″]

а.

;
г.

[/ hidden-answer]

Лебедь на озере поднимается в воздух, взмахивая крыльями и бегая по воде. (a) Если лебедь должен достичь скорости 6,00 м / с, чтобы взлететь, и он ускоряется из состояния покоя со средней скоростью

, как далеко он пролетит, прежде чем взлетит? б) Сколько времени это займет?

Мозг дятла особенно защищен от сильных ускорений связками внутри черепа, похожими на сухожилия.При клевании дерева голова дятла останавливается с начальной скорости 0,600 м / с на расстоянии всего 2,00 мм. (a) Найдите ускорение в метрах в секунду в квадрате и кратно g , где g = 9,80 м / с 2 . (b) Рассчитайте время остановки. (c) Сухожилия, удерживающие мозг, растягиваются, делая его тормозной путь 4,50 мм (больше, чем голова и, следовательно, меньше ускорение мозга). Каково ускорение мозга, кратное g ?

[show-answer q = ”fs-id1168326955141 ″] Показать решение [/ show-answer]

[скрытый-ответ a = ”fs-id1168326955141 ″]

а.

г.

;

г.

[/ hidden-answer]

Неосторожный футболист сталкивается со стойкой ворот с мягкой подкладкой при беге со скоростью 7,50 м / с и полностью останавливается, сжав подушку и свое тело на 0,350 м. а) Каково его ускорение? б) Как долго длится столкновение?

Посылка выпадает из грузового самолета и приземляется в лесу. Если предположить, что скорость посылки при ударе составляет 54 м / с (123 мили в час), то каково ее ускорение? Предположим, деревья и снег останавливают его на расстоянии 3.0 мин.

[show-answer q = ”fs-id1168326

9 ″] Показать решение [/ show-answer]

[скрытый-ответ a = ”fs-id1168326

9 ″]

Знает:

. Нам нужны a , поэтому мы можем использовать это уравнение:

.
[/ hidden-answer]

Скоростной поезд проходит через станцию. Он входит с начальной скоростью 22,0 м / с и замедляется со скоростью

м / с.

как проходит.Длина станции 210,0 м. а) Как быстро он движется, когда нос покидает станцию? б) Какова длина носа поезда на станции? (c) Если длина поезда 130 м, какова скорость конца поезда, когда он уезжает? (d) Когда поезд отправляется со станции?

Неоправданные результаты Драгстеры могут развить максимальную скорость 145,0 м / с всего за 4,45 с. (а) Рассчитайте среднее ускорение для такого драгстера. (b) Найдите конечную скорость этого драгстера, начиная с состояния покоя и ускоряясь со скоростью, найденной в (a) для 402.0 м (четверть мили) без использования информации о времени. (c) Почему конечная скорость больше той, которая используется для определения среднего ускорения? ( Подсказка : подумайте, справедливо ли предположение о постоянном ускорении для драгстера. Если нет, обсудите, будет ли ускорение больше в начале или в конце пробега и как это повлияет на конечную скорость.)

[показывать-ответ q = ”fs-id1168329316432 ″] Показать решение [/ показывать-ответ]

[скрытый-ответ a = ”fs-id1168329316432 ″]

а.

;
г.

;

г.

, потому что предположение о постоянном ускорении для драгстера неверно. Драгстер переключает передачи и будет иметь большее ускорение на первой передаче, чем на второй, чем на третьей, и так далее. Вначале ускорение будет максимальным, поэтому на

не будет.

за последние несколько метров, но существенно меньше, и конечная скорость будет меньше

.

.

[/ hidden-answer]

Глоссарий

задача преследования двух тел
задача кинематики, в которой неизвестные вычисляются путем одновременного решения кинематических уравнений для двух движущихся объектов.

Прямолинейное движение частиц — определение, вывод формул и примеры

Что такое прямолинейное движение?

Если положение объекта меняется относительно времени и его окружения, то говорят, что тело находится в движении.Математически движение можно описать смещением, скоростью и ускорением в конкретной системе отсчета. Движение частицы можно классифицировать на основе ее траектории, простейшим из которых является движение по прямой, а именно прямолинейное движение. Векторы смещения, скорости и ускорения ограничены одним измерением. Прямолинейное движение бывает трех типов: равномерное движение (нулевое ускорение), равномерно ускоренное движение (ненулевое постоянное ускорение) и движение с неравномерным ускорением.Примерами прямолинейного движения являются свободное падение под действием силы тяжести и простое гармоническое движение массы, прикрепленной к пружине.

Определение прямолинейного движения

Если частица ограничена движением по прямой линии, ее движение называется прямолинейным (или линейным) движением. Такое движение можно описать только одной координатой. Смещение частицы и ее производные, то есть скорость и ускорение, являются одномерными векторами. Свободное падение под действием гравитационного поля Земли, автомобиль, движущийся по прямой траектории, можно представить как прямолинейное движение.

[Изображение будет загружено в ближайшее время]

Математическая форма движения

Для качественного изучения прямолинейного движения используется одномерная система отсчета, состоящая из оси (ось X) и начала координат в точке O (x = 0). считается.

Положение, расстояние и смещение: Положение частицы — это векторная величина, которая указывает от начала координат до частицы. Его величина определяется расстоянием между ними. Когда частица приводится в движение, она следует по траектории, так что положение меняется со временем.Смещение — это разность векторов положения через некоторый промежуток времени, указывающая от исходного положения к конечному положению. Расстояние — это полный путь, пройденный по траектории, тогда как смещение — это кратчайший путь. Если положение частицы меняется с xi на xf во времени \ [\ Delta t \], смещение определяется как,

\ [x = {x_f} — {x_i} \]

Speed ​​and Velocity: Rate of изменение расстояния называется скоростью, а скорость изменения смещения во времени называется скоростью.2}}} x \]

Поскольку все векторы ограничены одним измерением, достаточно рассматривать только величины.

Графическое представление

Если положение отображается как функция времени, график показывает траекторию частицы. Скорость в любой момент задается наклоном этого графика, поскольку скорость является производной по времени от положения. Ускорение — это производная от скорости по времени, поэтому оно определяется наклоном графика зависимости скорости от времени.

Вывод формул прямолинейного движения

Учитывая различные значения ускорения, прямолинейное движение можно разделить на три типа: равномерное прямолинейное движение, равномерно ускоренное движение и движение с неравномерным ускорением.t {dt} \]

\ [x \ left (t \ right) {x_o} + {v_o} t \]

Здесь x₀ — начальное положение частицы, т.е. \ [x \ left ({t = 0} \ right) = {x_o} \] Ясно, что траектория частицы представляет собой прямую линию с постоянным наклоном v₀ и точкой пересечения y с x₀. Графики положение-время и скорость-время показаны ниже.

[Изображение будет загружено в ближайшее время]

\ [\ frac {{dv}} {{dt}} = {a_o} \]

Начальная скорость и смещение равны \ [{v_o} {\ text {and} } {x_o} \] соответственно i.2 + 2 {a_0} \ left ({x — {x_0}} \ right) \]

Это уравнение связывает положение и скорость в любой произвольный момент времени. Поскольку ускорение постоянно во времени, его можно представить как прямую линию, параллельную оси времени. Скорость также линейна, но меняется со временем, так что это прямая линия с ненулевым наклоном. Смещение квадратично по времени, а траектория параболическая.

[Изображение будет загружено в ближайшее время]

[Изображение будет загружено в ближайшее время]

Пример

Свободное падение под действием силы тяжести: если гравитационное ускорение \ [g \] объекта из-за гравитационного притяжения Земли считается равным постоянным на интересующем расстоянии, свободное падение объекта в гравитационном поле Земли можно представить как прямолинейное движение с постоянным ускорением. 2} \]

Это смещение от начальной высоты вниз, так что высота объекта уменьшается со временем.

Знаете ли вы?

  • Движение двух частиц под действием центральной силы (например, электростатической силы) можно представить как прямолинейное движение.

  • Свободное падение под действием гравитационного поля Земли на самом деле не является прямолинейным движением из-за вращения Земли. Сила Кориолиса из-за вращения вызывает искривление траектории свободного падения.

  • Линейное движение и вращение (на плоскости) вокруг оси имеют схожую динамику.

3.6 Определение скорости и смещения по ускорению

Цели обучения

К концу этого раздела вы сможете:

  • Выведите кинематические уравнения для постоянного ускорения с помощью интегрального исчисления.
  • Используйте интегральную формулировку кинематических уравнений при анализе движения.
  • Найдите функциональную форму зависимости скорости от времени с учетом функции ускорения.
  • Найдите функциональную форму зависимости положения от времени с учетом функции скорости.

В этом разделе предполагается, что у вас достаточно знаний в области вычислений, чтобы быть знакомыми с интеграцией. В разделах «Мгновенная скорость и скорость», «Среднее и мгновенное ускорение» мы ввели кинематические функции скорости и ускорения с использованием производной. Взяв производную функции положения, мы нашли функцию скорости, и аналогичным образом взяв производную функции скорости, мы нашли функцию ускорения. Используя интегральное исчисление, мы можем работать в обратном направлении и вычислять функцию скорости из функции ускорения и функцию положения из функции скорости.

Кинематические уравнения из интегрального исчисления

Начнем с частицы с ускорением a (t) — известная функция времени. Поскольку производной функции скорости по времени является ускорение,

[латекс] \ frac {d} {dt} v (t) = a (t), [/ латекс]

мы можем взять неопределенный интеграл от обеих сторон, найдя

[латекс] \ int \ frac {d} {dt} v (t) dt = \ int a (t) dt + {C} _ {1}, [/ latex]

, где C 1 — постоянная интегрирования.Поскольку [latex] \ int \ frac {d} {dt} v (t) dt = v (t) [/ latex], скорость определяется как

[латекс] v (t) = \ int a (t) dt + {C} _ {1}. [/ латекс]

Аналогично, производная по времени функции положения является функцией скорости,

[латекс] \ frac {d} {dt} x (t) = v (t). [/ латекс]

Таким образом, мы можем использовать те же математические манипуляции, которые мы только что использовали, и найти

[латекс] x (t) = \ int v (t) dt + {C} _ {2}, [/ latex]

, где C 2 — вторая постоянная интегрирования.

Используя эти интегралы, мы можем вывести кинематические уравнения для постоянного ускорения. Имея a ( t ) = a a константа, и выполняя интегрирование в (рисунок), мы находим

[латекс] v (t) = \ int adt + {C} _ {1} = at + {C} _ {1}. [/ латекс]

Если начальная скорость v (0) = v 0 , то

[латекс] {v} _ {0} = 0 + {C} _ {1}. [/ латекс]

Тогда C 1 = v 0 и

[латекс] v (t) = {v} _ {0} + at, [/ latex]

, который является (Уравнение).{2} [/ латекс]. а) Какова функция скорости моторной лодки? (б) В какое время скорость достигает нуля? (c) Какова функция местоположения моторной лодки? (d) Каково смещение моторной лодки с момента начала замедления до момента, когда скорость равна нулю? (e) Постройте график функций скорости и положения.

Стратегия

(a) Чтобы получить функцию скорости, мы должны интегрировать и использовать начальные условия, чтобы найти постоянную интегрирования. (b) Мы устанавливаем функцию скорости равной нулю и решаем для t . {2} + {C} _ {1}.{3} = 21.1 \, \ text {m} \ text {.} [/ Latex]

Рис. 3.30 (a) Скорость катера как функция времени. Катер снижает скорость до нуля за 6,3 с. Иногда скорость становится отрицательной — это означает, что лодка меняет направление. (b) Положение моторной лодки как функция времени. В момент времени t = 6,3 с скорость равна нулю, и лодка остановилась. В разы больше, чем это значение, скорость становится отрицательной — это означает, что если лодка продолжает двигаться с тем же ускорением, она меняет направление на противоположное и направляется обратно к месту своего зарождения.

Значение

Функция ускорения линейна по времени, поэтому интегрирование включает простые полиномы. На (Рисунок) мы видим, что если мы продолжим решение за точку, когда скорость равна нулю, скорость станет отрицательной, и лодка изменит направление на противоположное. Это говорит нам о том, что решения могут предоставить нам информацию, выходящую за рамки наших непосредственных интересов, и мы должны быть осторожны при их интерпретации. {2} [/ latex].{3}. [/ латекс]

  • Скорость может быть записана как v ( t ) = 5 t (1 — t ), что равно нулю при t = 0 и t = 1 с.
  • 2.5 Уравнения движения для постоянного ускорения в одном измерении — College Physics: OpenStax

    Мы могли бы знать, что чем больше ускорение, скажем, у автомобиля, удаляющегося от знака «Стоп», тем больше смещение за заданный промежуток времени. Но мы не разработали конкретное уравнение, связывающее ускорение и смещение.В этом разделе мы разработаем некоторые удобные уравнения для кинематических отношений, начиная с уже рассмотренных определений перемещения, скорости и ускорения.

    Сначала сделаем несколько упрощений в обозначениях. Принятие начального времени равным нулю, как если бы время измерялось секундомером, является большим упрощением. Поскольку прошедшее время равно [латекс] \ boldsymbol {\ Delta {t} = {t_f} — {t_0}} [/ latex], использование [latex] \ boldsymbol {t_0 = 0} [/ latex] означает, что [latex] \ boldsymbol {\ Delta {t} = {t_f}} [/ latex], последнее время на секундомере.Когда начальное время принимается равным нулю, мы используем индекс 0 для обозначения начальных значений положения и скорости. То есть [latex] \ boldsymbol {x_0} [/ latex] — это начальная позиция , а [latex] \ boldsymbol {v_0} [/ latex] — начальная скорость . Мы не ставим индексы на окончательные значения. То есть [latex] \ boldsymbol {t} [/ latex] — это последнее время , [latex] \ boldsymbol {x} [/ latex] — это конечное положение и [latex] \ boldsymbol {v } [/ latex] — конечная скорость .Это дает более простое выражение для истекшего времени — теперь [latex] \ boldsymbol {\ Delta {t} = t} [/ latex]. Это также упрощает выражение для смещения, которое теперь имеет вид [latex] \ boldsymbol {\ Delta {x} = {x} — {x_0}} [/ latex]. Кроме того, он упрощает выражение для изменения скорости, которое теперь имеет вид [latex] \ boldsymbol {\ Delta {v} = {v} — {v_0}} [/ latex]. Подводя итог, используя упрощенные обозначения, с начальным временем, принятым равным нулю,

    [латекс] \ begin {array} {lcl} \ boldsymbol {\ Delta {t}} & = & \ boldsymbol {t} \\ \ boldsymbol {\ Delta {x}} & = & \ boldsymbol {x-x_0} \\ \ boldsymbol {\ Delta {v}} & = & \ boldsymbol {v-v_0} \ end {array} [/ latex] [латекс] \ rbrace [/ latex]

    , где нижний индекс 0 обозначает начальное значение, а отсутствие нижнего индекса означает конечное значение в любом рассматриваемом движении.

    Теперь мы делаем важное предположение, что ускорение постоянно . Это предположение позволяет нам избегать использования расчетов для определения мгновенного ускорения. Поскольку ускорение постоянно, среднее и мгновенное ускорения равны. То есть

    [латекс] \ boldsymbol {\ bar {a} = a = \ textbf {constant,}} [/ latex]

    , поэтому мы всегда используем символ [latex] \ textbf {a} [/ latex] для ускорения. Предположение, что ускорение является постоянным, не серьезно ограничивает ситуации, которые мы можем изучить, и не ухудшает точность нашего лечения.Во-первых, ускорение равно постоянно в большом количестве ситуаций. Кроме того, во многих других ситуациях мы можем точно описать движение, приняв постоянное ускорение, равное среднему ускорению для этого движения. Наконец, в движениях, где ускорение резко меняется, например, когда автомобиль разгоняется до максимальной скорости, а затем тормозит до остановки, движение можно рассматривать в отдельных частях, каждая из которых имеет собственное постоянное ускорение.

    РЕШЕНИЕ ДЛЯ СМЕЩЕНИЯ (Δ

    x ) И КОНЕЧНОГО ПОЛОЖЕНИЯ ( x ) ОТ СРЕДНЕЙ СКОРОСТИ ПРИ УСКОРЕНИИ ( a ) ПОСТОЯННО

    Чтобы получить наши первые два новых уравнения, мы начнем с определения средней скорости:

    [латекс] \ boldsymbol {\ bar {v} =} [/ latex] [латекс] \ boldsymbol {\ frac {\ Delta {x}} {\ Delta {t}}} [/ латекс].

    Замена упрощенного обозначения для [latex] \ boldsymbol {\ Delta {x}} [/ latex] и [latex] \ boldsymbol {\ Delta {t}} [/ latex] дает

    [латекс] \ boldsymbol {\ bar {v} =} [/ latex] [латекс] \ boldsymbol {\ frac {x-x_0} {t}} [/ latex].

    Решение для [latex] \ boldsymbol {x} [/ latex] дает

    [латекс] \ boldsymbol {x = x_0 + \ bar {v} t} [/ latex],

    , где средняя скорость

    [латекс] \ boldsymbol {\ bar {v} =} [/ latex] [латекс] \ boldsymbol {\ frac {v_0 + v} {2}} [/ latex] [латекс] \ boldsymbol {(\ textbf {константа } \; а)} [/ латекс].

    Уравнение [латекс] \ boldsymbol {\ bar {v} =} [/ latex] [latex] \ boldsymbol {\ frac {{v} _0 + {v}} {2}} [/ latex] отражает тот факт, что, когда ускорение постоянное, [latex] \ boldsymbol {v} [/ latex] — это просто среднее значение начальной и конечной скорости. Например, если вы постоянно увеличиваете свою скорость (то есть с постоянным ускорением) с 30 до 60 км / ч, то ваша средняя скорость во время этого постоянного увеличения составляет 45 км / ч. Используя уравнение [латекс] \ boldsymbol {\ bar {v} =} [/ latex] [latex] \ boldsymbol {\ frac {{v} _0 + {v}} {2}} [/ latex], чтобы проверить это, мы смотри, что

    [латекс] \ boldsymbol {\ bar {v} =} [/ latex] [латекс] \ boldsymbol {\ frac {{v} _0 + {v}} {2}} [/ latex] [латекс] \ boldsymbol { =} [/ latex] [латекс] \ boldsymbol {\ frac {30 \ textbf {км / ч} + 60 \ textbf {км / ч}} {2}} [/ latex] [латекс] \ boldsymbol {= 45 \ textbf {km / h}} [/ latex],

    , что кажется логичным.

    Пример 1: Расчет смещения: как далеко пробегает бегунок?

    Бегун бежит по прямому участку дороги со средней скоростью 4,00 м / с в течение 2,00 мин. Какова его конечная позиция, если исходная позиция равна нулю?

    Стратегия

    Нарисуйте эскиз.

    Рис. 2.

    Конечная позиция [латекс] \ boldsymbol {x} [/ latex] определяется уравнением

    [латекс] \ boldsymbol {{x} = {x} _0 + \ bar {v} {t}} [/ latex].

    Чтобы найти [latex] \ boldsymbol {x} [/ latex], мы определяем значения [latex] \ boldsymbol {x_0} [/ latex], [latex] \ boldsymbol {\ bar {v}} [/ latex] , и [latex] \ boldsymbol {t} [/ latex] из постановки задачи и подставьте их в уравнение.

    Решение

    1. Определите известные. [Latex] \ boldsymbol {\ bar {v} = 4.00 \ textbf {m / s}} [/ latex], [latex] \ boldsymbol {\ Delta {t} = 2.00 \ textbf {min }} [/ latex] и [latex] \ boldsymbol {{x} _0 = 0 \ textbf {m}} [/ latex].

    2. Введите известные значения в уравнение.

    [латекс] \ boldsymbol {{x} = {x} _0 + \ bar {v} {t} = \: 0 + \ 🙁 4.00 \ textbf {m / s}) (120 \ textbf {s}) = 480 \ textbf {m}} [/ latex]

    Обсуждение

    Скорость и конечное смещение положительны, что означает, что они направлены в одном направлении.2} [/ латекс]. На графике линейные функции выглядят как прямые линии с постоянным наклоном.) Например, в автомобильной поездке мы продвинемся вдвое дальше за заданное время, если мы усредним 90 км / ч, чем если бы мы в среднем 45 км / ч.

    Рис. 3. Между смещением и средней скоростью существует линейная зависимость. В течение заданного времени t объект, движущийся в два раза быстрее другого объекта, будет перемещаться вдвое дальше другого объекта.

    РЕШЕНИЕ ДЛЯ КОНЕЧНОЙ СКОРОСТИ


    Мы можем вывести еще одно полезное уравнение, манипулируя определением ускорения.

    [латекс] \ boldsymbol {{a} =} [/ latex] [латекс] \ boldsymbol {\ frac {\ Delta {v}} {\ Delta {t}}} [/ latex]

    Замена упрощенного обозначения для [latex] \ boldsymbol {\ Delta {v}} [/ latex] и [latex] \ boldsymbol {\ Delta {t}} [/ latex] дает нам

    [латекс] \ boldsymbol {a =} [/ latex] [латекс] \ boldsymbol {\ frac {{v} — {v} _0} {t}} [/ latex] [латекс] \ boldsymbol {(\ textbf { константа} а)} [/ латекс].

    Решение для [latex] \ boldsymbol {v} [/ latex] дает

    [латекс] \ boldsymbol {{v} = {v} _0 + at \: (\ textbf {constant} a)} [/ latex].2) (40.0 \ textbf {s}) = 10.0 \ textbf {m / s}} [/ latex]

    Обсуждение

    Конечная скорость намного меньше начальной скорости, требуемой при замедлении, но все же положительная. С помощью реактивных двигателей обратная тяга могла поддерживаться достаточно долго, чтобы остановить самолет и начать движение назад. На это указывает отрицательная конечная скорость, чего здесь нет.

    Рис. 5. Самолет приземляется с начальной скоростью 70,0 м / с и замедляется до конечной скорости 10.0 м / с до направления к терминалу. Обратите внимание, что ускорение отрицательное, потому что его направление противоположно его скорости, которая положительна.

    Уравнение [latex] \ boldsymbol {v = v_0 + at} [/ latex] не только помогает при решении задач, но и дает нам представление о взаимосвязях между скоростью, ускорением и временем. Из него видно, например, что

    • Конечная скорость зависит от того, насколько велико ускорение и как долго оно длится
    • , если ускорение равно нулю, то конечная скорость равна начальной скорости [латекс] \ boldsymbol {(v = v_0)} [/ latex], как и ожидалось (т.е., скорость постоянна)
    • если [latex] \ boldsymbol {a} [/ latex] отрицательно, то конечная скорость меньше начальной скорости

    (Все эти наблюдения соответствуют нашей интуиции, и всегда полезно исследовать основные уравнения в свете нашей интуиции и опыта, чтобы убедиться, что они действительно точно описывают природу.)

    ПОДКЛЮЧЕНИЕ: СОЕДИНЕНИЕ В РЕАЛЬНОМ МИРЕ


    Рис. 6. Космический шаттл Endeavour стартует из Космического центра Кеннеди в феврале 2010 года.(Источник: Мэтью Симантов, Flickr).

    Межконтинентальная баллистическая ракета (МБР) имеет большее среднее ускорение, чем космический шаттл, и достигает большей скорости в первые две минуты полета (фактическое время горения межконтинентальной баллистической ракеты засекречено — ракеты с коротким временем горения сложнее для противника. разрушать). Но космический шаттл получает большую конечную скорость, так что он может вращаться вокруг Земли, а не сразу возвращаться вниз, как это делает межконтинентальная баллистическая ракета. Космический шаттл делает это за счет более длительного ускорения.

    РЕШЕНИЕ ДЛЯ КОНЕЧНОГО ПОЛОЖЕНИЯ, КОГДА СКОРОСТЬ НЕ ПОСТОЯННА (a ≠ 0)

    Мы можем объединить приведенные выше уравнения, чтобы найти третье уравнение, которое позволяет нам вычислить окончательное положение объекта, испытывающего постоянное ускорение. Начнем с

    [латекс] \ boldsymbol {v = v_0 + at} [/ латекс].

    Добавление [latex] \ boldsymbol {v_0} [/ latex] к каждой стороне этого уравнения и деление на 2 дает

    [латекс] \ boldsymbol {\ frac {v_0 + \: v} {2}} [/ latex] [latex] \ boldsymbol {= v_0 +} [/ latex] [латекс] \ boldsymbol {\ frac {1} {2} } [/ latex] [латекс] \ boldsymbol {at} [/ latex]

    Поскольку [latex] \ boldsymbol {\ frac {v_0 + v} {2} = \ bar {v}} [/ latex] для постоянного ускорения, то

    [латекс] \ boldsymbol {\ bar {v} = v_0 +} [/ latex] [латекс] \ boldsymbol {\ frac {1} {2}} [/ latex] [латекс] \ boldsymbol {at} [/ latex] .2} [/ латекс]. Предположим, такой драгстер ускоряется из состояния покоя за 5,56 с. Как далеко он пролетит за это время?

    Рис. 7. Пилот Top Fuel американской армии Тони «Сержант» Шумахер начинает гонку с контролируемого выгорания. (Источник: подполковник Уильям Термонд. Фотография предоставлена ​​армией США).

    Стратегия

    Нарисуйте эскиз.

    Рисунок 8.

    Нас просят найти смещение, которое равно [latex] \ boldsymbol {x} [/ latex], если принять [latex] \ boldsymbol {x_0} [/ latex] равным нулю.2} [/ latex],

    дает

    [латекс] \ boldsymbol {x = 402 \ textbf {m}} [/ латекс].

    Обсуждение

    Если мы переведем 402 м в мили, мы обнаружим, что пройденное расстояние очень близко к четверти мили, стандартному расстоянию для дрэг-рейсинга. Так что ответ разумный. Это впечатляющее смещение всего за 5,56 с, но первоклассные драгстеры могут проехать четверть мили даже за меньшее время. 2} [/ latex].2 + 2a (x-x_0)} [/ latex] может дать дополнительную информацию об общих отношениях между физическими величинами:

    • Конечная скорость зависит от того, насколько велико ускорение и расстояние, на котором оно действует
    • При фиксированном замедлении автомобиль, который едет вдвое быстрее, не просто останавливается на удвоенном расстоянии — для остановки требуется гораздо больше времени. (Вот почему у нас есть зоны с пониженной скоростью возле школ.)

    Уравнения движения с постоянной скоростью

    Движение с постоянной скоростью , также известное как равномерное прямолинейное движение (u.среднеквадратичное значение) , это тот, у которого постоянная скорость , т.е. траектория прямолинейная, а скорость постоянная. В этом разделе мы собираемся объяснить:

    Определение движения с постоянной скоростью

    Хотя обнаружение прямолинейного равномерного движения или движения с постоянной скоростью в природе встречается довольно редко, его легче всего изучать, и оно будет полезно при изучении других, более сложных движений. Равномерное прямолинейное движение имеет следующие свойства:

    • Ускорение равно нулю ( a = 0 ), потому что ни величина, ни направление не меняются
    • С другой стороны, начальная, средняя и мгновенная скорости всегда имеют одинаковые значения

    Тело имеет движение с постоянной скоростью или равномерное прямолинейное движение , когда его траектория является прямой линией и его скорость постоянна .Это означает, что он преодолевает равные расстояния в равное время .

    Уравнения движения с постоянной скоростью

    Прямолинейное и равномерное движение

    Равное время тратится на путешествие на равные расстояния. Средняя скорость постоянна и равна величине скорости.

    Уравнения движения с постоянной скоростью :

    Где:

    • x , x 0 : Позиция тела в данный момент времени ( x ) и в начальный момент времени ( x 0 ).Единицей измерения в Международной системе (S.I.) является метр (м)
    • .
    • v , v 0 : Скорость тела в данный момент времени ( v ) и в начальный момент времени ( v 0 ). Его единица измерения в Международной системе (S.I.) — метр в секунду (м / с)
    • .
    • a : Ускорение кузова. Единицей измерения в Международной системе (S.I.) является метр на секунду в квадрате (м / с 2 )

    Для вывода уравнений равномерного прямолинейного движения u.среднеквадратичное значение следует учитывать, что:

    • Средняя скорость совпадает с мгновенной скоростью
    • Нет разгона

    С этими ограничениями получаем:

    vavg = vvavg = ΔxΔt = x-x0t-t0 = ⏟t0 = 0x-x0t → x-x0 = v⋅t → x = x0 + v⋅t

    Пример

    Два мраморных игрока смотрят друг на друга со своими шариками в руках. Игра состоит из одновременного бросания шариков по прямой линии так, чтобы они ударялись друг о друга.Игроки расположены в 36 метрах друг от друга, и игрок A запускает шарик со скоростью 2 м / с, а игрок B — со скоростью 4 м / с, в равномерном прямолинейном движении. Вычислите расстояние от игрока B, на котором шарики столкнутся.

    Физический тест на «прямолинейное равномерное и равноускоренное движение». Прямолинейное равномерное движение Тс 1 вариант прямолинейного равномерного движения

    Существуют различные типы механического движения. В зависимости от формы траектории движение может быть прямым или криволинейным.При движении скорость тела может оставаться постоянной или изменяться со временем. В зависимости от характера изменения скорости движение будет равномерным или неравномерным.

    Прямолинейное движение — это движение, при котором траектория тела (точек) представляет собой прямую линию. Например, движение автомобиля по участку дороги, на котором нет подъемов, спусков, поворотов.

    Равномерное прямолинейное движение — это движение, при котором тело проходит одни и те же траектории через любые равные промежутки времени и направление движения не меняется Я есть.

    Если сравнить равномерное движение нескольких тел, то можно отметить, что скорость изменения их положения в пространстве может быть разной, что характеризуется физической величиной, называемой скоростью.

    Скорость равномерного прямолинейного движения называется векторной физической величиной, равной отношению движения тела ко времени, в течение которого это движение произошло.

    (1)

    Единица измерения скорости в СИ — метр в секунду (1 м / c ).За единицу скорости принимается скорость такого равномерного движения, при которой тело 1 с перемещений 1 м .

    При прямом равномерном движении скорость не меняется со временем.

    Зная скорость равномерного движения, можно найти движение тела за любой промежуток времени:

    (2)

    При равномерном прямолинейном движении векторы скорости и смещения направлены в одном направлении.

    Основная задача механики — определить положение тела в любой момент времени, то есть определить его координаты. Уравнение движения — это зависимость координат тела от времени при равномерном прямолинейном движении.

    Тело переместилось … Направим ось X в направлении движения тела. х 0 — начальная координата тела, x — конечная координата тела.

    Таким образом, координата тела с равномерным прямолинейным движением в любой момент времени может быть определена, если известны его начальная координата и проекция скорости движения на ось NS … Проекции скорости и смещения могут быть как положительными, так и отрицательными.

    График зависимости модуля вектора скорости от времени при равномерном движении представляет собой прямую, параллельную оси абсцисс. Действительно, с течением времени скорость при таком движении остается постоянной.

    График зависимости скорости тела от времени при равномерном движении V = const

    При прямолинейном равномерном движении модуль вектора смещения численно равен площади под графиком смещения к оси времени.

    График зависимости движения тела от времени при прямолинейном равномерном движении представляет собой прямую, проходящую через начало координат. Причем, чем круче график движения, тем больше скорость тела.

    График зависимости пройденного телом пути от времени

    При прямолинейном равномерном движении модуль вектора скорости численно равен тангенсу угла наклона графика смещения к оси времени.

    Поскольку зависимость координат тела от времени является линейной функцией, соответствующий график зависимости (график движения) представляет собой прямую линию. Пример построения такого графика показан на рисунке.

    График зависимости координат тела от времени

    Это руководство включает обучающие задания. тесты на самоконтроль, самостоятельную работу, контрольную работу и примеры решения типовых задач. Предлагаемые дидактические материалы составлены в полном соответствии со структурой и методикой учебника А.В. Перышкин, К. М. Гутник «Физика. Оценка 9 «.

    TK-1. Путь и движение .
    1. Укажите, в каком из следующих примеров тело можно рассматривать как материальную точку:
    a) Земля движется вокруг Солнца;
    b) Земля вращается на своей оси;
    в) Луна, вращающаяся вокруг Земли;
    г) Луна, по поверхности которой движется луноход;
    д) молот, брошенный спортсменом;
    е) спортивный молот, который изготовлен на станке.
    2.Что определяет пассажира автобуса по номерам на километровых столбах, установленных вдоль трассы — движение или расстояние, пройденное автобусом?
    3. На рисунке 1 показаны траектории полета снаряда. Равны ли пути, пройденные снарядами, для этих движений? смещение?
    4. Тело, брошенное вертикально вверх из точки L, упало в шахту (рис. 2). Какое расстояние пройдено телом и модуль движения, если AB = 15 м, BC — 18 м?
    5. Спортсмен должен пробежать один круг (400 м).Каков модуль смещения, если он: а) прошел 200 м пути; б) закончили? Рассмотрим дорожку стадиона как круг.
    6. Белка бежит внутри колеса, находясь на одинаковой высоте относительно пола. Равны ли путь и смещение для такого движения?

    Предисловие.
    УЧЕБНЫЕ ЗАДАНИЯ
    ТК-1. Путь и движение.
    ТК-2. Прямолинейное равномерное движение.
    ТК-3. Относительность движения.
    ТК-4. Прямолинейное равноускоренное движение.
    ТК-5. Законы Ньютона.
    ТК-6. Свободное падение тел.
    ТК-7. Закон всемирного тяготения. Кузовной механизм
    ТК-8. Импульс тела. Закон сохранения импульса.
    Закон сохранения энергии.
    ТК-9. Механические колебания и волны. Звук.
    ТК-10. Электромагнитное поле.
    ТК-11. Строение атома и атомного ядра.
    ИСПЫТАНИЯ ДЛЯ САМОКОНТРОЛЯ
    ТС-1. Прямолинейное равномерное движение.
    ТС-2. Прямолинейное равноускоренное движение.
    ТС-3. Законы Ньютона.
    ТС-4. Свободное падение тел.
    ТС-5. Закон всемирного тяготения. Движение тела
    по окружности. Искусственные спутники Земли ..
    ТС-6. Импульс тела. Закон сохранения импульса.
    Закон сохранения энергии.
    ТС-7. Механические колебания.
    ТС-8. Механические волны. Звук.
    ТС-9. Электромагнитное поле.
    ТС-10. Строение атома и атомного ядра.
    НЕЗАВИСИМЫЕ РАБОТЫ
    СР-1.Путь и движение.
    СР-2. Прямолинейное равномерное движение.
    СР-3. Прямолинейное равномерное движение.
    Графические задания.
    СР-4. Относительность движения.
    СР-5. Прямолинейное равноускоренное движение.
    СР-6. Прямолинейное равноускоренное движение.
    Графические задания.
    СР-7. Законы Ньютона.
    СР-8. Свободное падение тел.
    СР-9. Закон всемирного тяготения.
    Искусственные спутники Земли.
    СР-10. Движение тела по кругу.
    СР-11. Импульс тела. Закон сохранения импульса.
    Закон сохранения энергии.
    СР-12. Механические колебания.
    СР-13. Механические волны. Звук.
    СР-14. Электромагнитное поле.
    СР-15. Строение атома и атомного ядра.
    ТЕСТОВЫЕ ДОКУМЕНТЫ
    КР-1. Прямолинейное равноускоренное движение.
    КР-2. Законы Ньютона.
    КР-3. Закон всемирного тяготения. Движение тела
    по окружности. Искусственные спутники Земли.
    КР-4. Закон сохранения импульса.
    Закон сохранения энергии.
    КР-5. Механические колебания и волны.
    КР-6. Электромагнитное поле.
    ПРИМЕРЫ РЕШЕНИЯ ТИПОВЫХ ЗАДАЧ
    Законы взаимодействия и движения тел.
    Механические колебания и волны.
    Электромагнитное поле.
    ОТВЕТОВ
    Учебные задания.
    Самоконтроль.
    Самостоятельная работа.
    Контрольные работы.
    Библиография.

    Скачать бесплатно электронную книгу в удобном формате, смотреть и читать:
    Скачать книгу Физика, 9 класс, Учебное пособие, Марон А.Э., Марон Э.А., 2014 — fileskachat.com, скачать быстро и бесплатно.

    Скачать pdf
    Ниже вы можете купить эту книгу по лучшей сниженной цене с доставкой по всей России.

    Равномерное движение — это движение с постоянной скоростью, то есть когда скорость не изменяется (v = const) и не происходит ускорения или замедления (a = 0).

    Прямое движение — это движение по прямой, то есть траектория прямолинейного движения представляет собой прямую линию.

    Равномерное прямолинейное движение — Это движение, при котором тело совершает одни и те же движения в течение любых равных интервалов времени. Например, если мы разделим некоторый временной интервал на отрезки по одной секунде, то при равномерном движении тело будет перемещаться на одно и то же расстояние для каждого из этих отрезков времени.

    Скорость равномерного прямолинейного движения не зависит от времени и в каждой точке траектории направлена ​​так же, как движение тела. То есть вектор смещения совпадает по направлению с вектором скорости.В этом случае средняя скорость за любой период времени равна мгновенной скорости:

    V cp = v

    Пройденное расстояние при прямолинейном движении равно модулю смещения. Если положительное направление оси OX совпадает с направлением движения, то проекция скорости на ось OX равна величине скорости и положительна:

    V x = v, то есть v> 0

    Проекция смещения на ось OX равна:

    S = vt = x — x 0

    где x 0 — начальная координата тела, x — конечная координата тела (или координата кузов в любое время)

    Уравнение движения , то есть зависимость координат тела от времени x = x (t), принимает вид:

    X = x 0 + vt

    Если положительное направление оси OX противоположно относительно направления движения тела, то проекция скорости тела на ось OX отрицательна, скорость меньше нуля (v

    X = x 0 — vt

    Временная зависимость скорости, координат и пути

    Зависимость проекции скорости тела от времени показана на рис.1.11. Поскольку скорость постоянна (v = const), график скорости представляет собой прямую линию, параллельную оси времени Ot.

    Рис. 1.11. Зависимость проекции скорости тела от времени при равномерном прямолинейном движении.

    Проекция смещения на координатную ось численно равна площади прямоугольника OABS (рис. 1.12), поскольку величина вектора смещения равна произведению вектора скорости на время, в течение которого перемещение было произведено.

    Рис. 1.12. Зависимость проекции движения тела от времени при равномерном прямолинейном движении.

    График движения во времени показан на рис. 1.13. Из графика видно, что проекция скорости составляет

    V = s 1 / t 1 = tan α

    , где α — угол наклона графика к оси времени. Чем больше угол α, тем быстрее движется тело, то есть тем больше его скорость (чем дольше тело движется за меньшее время).Тангенс угла наклона касательной к графику зависимости координаты от времени равен скорости:

    Tg α = v

    Рис. 1.13. Зависимость проекции движения тела от времени при равномерном прямолинейном движении.

    Зависимость координаты от времени показана на рис. 1.14. Из рисунка видно, что

    Tg α 1> tg α 2

    , следовательно, скорость тела 1 выше скорости тела 2 (v 1> v 2).

    Tg α 3 = v 3

    Если тело находится в состоянии покоя, то график координат представляет собой прямую линию, параллельную оси времени, то есть

    X = x 0

    Рис. 1.14. Зависимость координат тела от времени при равномерном прямолинейном движении.

    М .: 2014. — 128с. М .: 2005. — 128с.

    Пособие включает обучающие задания, тесты на самоконтроль, самостоятельную работу, контрольную работу и примеры решения типовых задач.Предлагаемые дидактические материалы составлены в полном соответствии со структурой и методикой учебника А.В. Перышкин, М.Е. Гутник «Физика. 9 класс».

    Формат: pdf ( 2014 г. , 128с.)

    Размер: 2,8 Мб

    Часы, скачать: 02

    Формат: pdf ( 2005 г. , 128с.)

    Размер: 6,8 Мб

    Загрузить: 02 .09.2016, ссылки удалены по просьбе ИД «Дрофа» (см. Примечание)

    Содержание
    Предисловие 3
    УЧЕБНЫЕ ЗАДАНИЯ
    ТК-1.Путь и проезд 5
    ТК-2. Прямолинейный равномерный 6
    ТК-3. Относительность движения 8
    ТК-4. Прямолинейное равноускоренное движение 10
    ТК-5. Законы Ньютона 13
    ТК-6. Свободнопадающие тела 16
    TK-7. Закон всемирного тяготения. Движение тела по кругу. Искусственные спутники Земли 17
    ТК-8. Импульс тела. Закон сохранения импульса 19
    TK-9. Механические колебания и волны. Звук 20
    ТК-10. Электромагнитное поле 22
    ТК-11.Строение атома и атомного ядра 24
    ТЕСТЫ ДЛЯ САМОКОНТРОЛЯ
    TS-1. Прямолинейный равномерный механизм 25
    ТС-2. Прямолинейный равноускоренный механизм 28
    ТС-3. Законы Ньютона 31
    ТУ-4. Свободно падающие тела 34
    TS-5. Закон всемирного тяготения. Движение тела по кругу. Искусственные спутники Земли. … 35
    ТС 6. Тело импульс. Закон сохранения количества движения 38
    ТС-7. Механические колебания 39
    ТС-8.Механические волны. Звук 42
    ТС-9. Электромагнитное поле 45
    ТС-10. Строение атома и атомного ядра 48
    НЕЗАВИСИМЫЕ РАБОТЫ
    CP-1. Путь и путешествие 52
    СР-2. Прямолинейный равномерный механизм 55
    СР-3. Прямолинейное равномерное движение. Графические задания 58
    СР-4. Относительность движения 61
    CP-5. Прямолинейное равноускоренное движение 64
    СР-6. Прямолинейное равноускоренное движение. Графические задания 66
    СР-7. Законы Ньютона 71
    CP-8.Свободно падающие тела 73
    CP-9. Закон всемирного тяготения. Искусственные спутники Земли 74
    СР-10. Движение тела по кругу 75
    СР-11. Импульс тела. Закон сохранения количества движения 77
    CP-12. Механические колебания 79
    СР-13. Механические волны. Звук G 80
    CP-14. Электромагнитное поле 82
    СР-15. Строение атома и атомного ядра 86
    ТЕСТОВЫЕ ДОКУМЕНТЫ
    КР-1. Прямолинейное равноускоренное движение 89
    КР-2.Законы Ньютона 93
    КР-3. Закон всемирного тяготения. Движение тела по кругу. Искусственные спутники Земли 97
    КР-4. Закон сохранения количества движения 101
    КР-5. Механические колебания и волны 105
    КР-6. Электромагнитное поле 109
    ПРИМЕРЫ РЕШЕНИЯ ТИПОВЫХ ЗАДАЧ
    Законы взаимодействия и движения тел 113
    Механические колебания и волны 117
    Электромагнитное поле 118
    ОТВЕТЫ
    Практические задания 119
    Самопроверка 120
    Самостоятельная работа 121
    Испытательные работы 124
    Список литературы 126

    Пособие включает обучающие задания (ТЗ), тесты на самоконтроль (TS), самостоятельную работу (SR), контрольную работу (CR), примеры решения типовых задач.
    Учебный комплект предусматривает организацию всех основных этапов учебно-познавательной деятельности школьников в соответствии с требованиями ФГОС: применение и актуализация теоретических знаний, самоконтроль качества усвоения знаний. материал, использование алгоритмов решения задач, выполнение самостоятельных и контрольно-оценочных работ.
    Учебные задания (ТК 1-11) для всех разделов 9-го класса курса физики содержат комплекс качественных экспериментальных и графических заданий, ориентированных на формирование ведущих понятий и основных закономерностей курса.Задания подобраны таким образом, чтобы дать студенту возможность осмыслить существенные особенности концепции, рассмотреть физическое явление на уровне фактов, физических величин и физических законов. Авторы стремились оформить учебные задания в виде небольшого сборника задач, дополняющего систему типовых заданий учебника и позволяющего организовать дифференцированное аудиторное и домашнее задание.
    Тесты на самоконтроль (ТС 1-10) с выбором ответа предназначены для оперативного, тематического контроля на основе урока и самоконтроля знаний.В зависимости от конкретных условий (подготовка к уроку, организация многоуровневого обучения и др.) Преподаватель может варьировать набор тестовых заданий и определять время их выполнения.
    Самостоятельная работа (СР 1-15) содержит 10 вариантов и рассчитана примерно на 20 минут каждый. Для дифференциации обучения более подготовленным ученикам рекомендуется совмещать варианты 7 и 8; 9 и 10.

    2.1: Равномерное линейное движение — Physics LibreTexts

    1. Последнее обновление
    2. Сохранить как PDF
    Без заголовков

    Линейное движение означает, что объект движется по прямой линии.«Равномерное» просто означает, что ускорение равно 0 на протяжении всего движения. Другими словами, скорость остается постоянной / равномерной. В виде графика

    По определению, скорость — это скорость пройденного расстояния. В этом простом случае

    скорость = расстояние / время

    Следовательно, по аналогии

    скорость = смещение / время

    Термин смещение — это просто векторный аналог направления.Однако важно отметить, что смещение дает только расстояние от начальной и конечной позиции, а не длину всего пути, как это было бы. Это станет ясно, когда мы обсудим векторы более подробно позже. Для иллюстрации представьте, что вы идете на 1 метр вперед, а затем на 1 метр назад. Хотя расстояние, которое вы пройдете, будет 2 м, поскольку ваше начальное и конечное положение одинаковы, ваше смещение равно 0. Отсюда смещение будет обозначено \ (\ overrightarrow {s} \), а скорость — \ ( \ overrightarrow {v} \).По соглашению, небольшая стрелка помещается вверху векторного значения или символ выделяется жирным шрифтом (например, v ).

    Итак,

    \ (\ overrightarrow {s} = \ overrightarrow {v} \ times t \ nonumber \) Путем перестановки

    На графике это соответствует области между осью x и линией, обозначающей скорость. Итак, если мы возьмем покрытую площадь от t = 0 до t = 5, , мы получим 20 метров.

    Добавить комментарий

    Ваш адрес email не будет опубликован. Обязательные поля помечены *